CAPITULO I SUPERFICIES: TEORÍA Y PROBLEMAS RECTAS Y PLANOS EN EL ESPACIO TRIDIMENCIONAL

CAPITULO I SUPERFICIES: TEORÍA Y PROBLEMAS RECTAS Y PLANOS EN EL ESPACIO TRIDIMENCIONAL El presente trabajo empieza con presentar las técnicas de la G

415 downloads 71 Views 3MB Size

Recommend Stories


PUNTOS, RECTAS Y PLANOS EN EL ESPACIO
6 PUNTOS, RECTAS Y PLANOS EN EL ESPACIO Página 153 REFLEXIONA Y RESUELVE Puntos alineados en el plano ■ Comprueba que los puntos A (5, 2), B (8, 3)

TEMA 7. PUNTOS, RECTAS Y PLANOS EN EL ESPACIO
TEMA 7 MATEMÁTICAS II TEMA 7. PUNTOS, RECTAS Y PLANOS EN EL ESPACIO. 1. INTRODUCCIÓN .............................................................

Puntos, rectas y planos
UNIDAD 5 Puntos, rectas y planos i en la Unidad anterior estudiamos los vectores y las operaciones con vectores, en ésta y en la siguiente estudiar

Story Transcript

CAPITULO I SUPERFICIES: TEORÍA Y PROBLEMAS RECTAS Y PLANOS EN EL ESPACIO TRIDIMENCIONAL El presente trabajo empieza con presentar las técnicas de la Geometría Espacial en el campo de la Ingeniería y trabajaremos usando una teoría adecuada de fácil entendimientos para estudiantes que recién empiezan a trabajar el calculo de varias variables de tal manera que sea este curso amigable y de fácil comprensión y que en forma autodidacta el alumno aprecie las bondades de todos los temas tratados, ya que el que sabe identificar la superficie y ubicarlo en el espacio de tres dimensiones podrá bosquejar su grafica y dar solución a los problemas que se les presente ya sea de, funciones vectoriales, funciones vectoriales de varias variables de máximos y mínimos, de integrales de línea y de las aplicaciones de integrales múltiples. En este trabajo se describe en forma detallada la teoría y ejercicios y problemas e implementamos algunos graficadores en el especio de tres dimensiones. La matemática actual y en especial el Cálculo se caracteriza por la importancia que le confiere a la Geometría Espacial y a las funciones de varias variables, por considerar que tanto las operaciones numéricas como las lógicas en las funciones de varias variables usando la Geometría Espacial representan procesos estrechamente ligados. Aquí sugerimos algunas características deseables del estudiante:  Habilidad para encontrar similitudes y relaciones entre cosas aparentemente distintas.  Facilidad para abstraer.  Tener pensamiento lógico y ordenado.  Que le guste aclarar las cosas hasta entenderlas perfectamente.  Profundizar en los temas que sean necesarios.  Perceverancia suficiente para trabajar en la resolución de los problemas que se le presenten hasta encontrar alguna solución.  Habilidad para analizar y construir.  Capacidad para generalizar. 1

1.

SISTEMA DE COORDENADA RECTANGULAR EN EL ESPACIO Consideremos tres planos mutuamente perpendiculares, Pxy, PXz, Pyz, que se cortan en un mismo punto O. En la figura identificamos los siguientes elementos geométricos.

a) EJES COORDENADOS.- Los ejes generalmente son identificados por letras X, Y, Z y se habla frecuentemente del eje X, del eje Y y del eje Z, donde: El eje X es la recta determinada por la intersección de los planos Pxy y Pxz, el eje Y es la recta determinada por la intersección de los planos Pxy y Pyz y El eje Z es la recta determinada por la intersección de los planos Pxz y Pyz. La dirección positiva se indica por medio de una flecha. Los ejes coordenados tomados de dos en dos determinan tres planos, llamados planos coordenados. b) PLANOS COORDENADOS.- El plano coordenado XY que denotaremos por Pxy, es determinado por las rectas: eje X y eje Y. El plano coordenado XZ que denotaremos por Pxz, es determinado por las rectas: eje X y eje Z. El plano coordenado YZ que denotaremos por Pyz, es determinado por las rectas: eje Y y eje Z. Los planos coordenados dividen al espacio tridimensional en 8 sub-espacios llamados octantes. Consideramos un punto p(x, y, z), cualquiera en el espacio tridimensional, a través de p(x, y, z) se construye tres planos un plano perpendicular a cada uno de los ejes coordenados. Sean A(x, 0, 0) el punto en el cual el plano perpendicular corta al eje X, B(0, y, 0) el punto en el cual el plano perpendicular corta al eje Y, y sea C(0, 0, z)el punto en el cual el plano perpendicular corta al eje Z

2

2.

DISTANCIA ENTRE DOS PUNTOS TEOREMA.-

La distancia no dirigida entre dos puntos p1(x1, y1, z1) y p2(x2, y2, z2) del espacio tridimensional está dado por:

d  p1 , p2  

 x2  x1    y2  y1    z2  z1  2

2

2

DEMOSTRACIÓN Sea a  p1 p2 un vector de origen p1 y extremo p2, entonces:

a  p1 p2  p2  p1   x2  x1 , y2  y1 , z2  z1  por lo tanto la longitud del vector a es:

d  p1 , p2  || a || 3.

 x2  x1    y2  y1    z2  z1  2

2

2

DIVISIÓN DE UN SEGMENTOSEGÚN RAZÓN DADA TEOREMA.-

Si los puntos p1(x1, y1, z1) y p2(x2, y2, z2) son extremos de un segmento dirigido; las coordenadas de un punto p(x, y, z) que divide al segmento p1 p2 en la Razón r  p1 p  pp2 es: x

x1  rx2 y  ry2 z  rz2 ,y 1 ,x  1 , r  1 1 r 1 r 1 r

DEMOSTRACIÓN Del gráfico se tiene: p1 p // pp2   r  R tal que: p1 p  r pp2 ,de donde 1 p  p1  r  p2  p  al despejar p se tiene: p   p1  rp2  , ahora reemplazamos 1 r por sus coordenadas respectivas: 1  x, y, z    x1, y1, z1   r  x2 , y2 , z2   1 r x  rx y  ry z  rz  x, y, z    1 2 , 1 2 , 1 2  , por igualdad 1 r 1 r   1 r se tiene:

x

x1  rx2 y  ry2 z  rz2 ,y 1 ,z  1 , r  1 1 r 1 r 1 r

3

COROLARIO.- Si p(x, y, z) es el punto medio segmento p1 p2 entonces r 

p1 p  1. pp2

Luego las coordenadas del punto medio son:

x 4.

x1  x2 y y z z , y  1 2 ,z  1 2 2 2 2

ÁNGULOS DIRECTORES, COSENO DIRECTORES Y NÚMEROS DIRECTORES Consideremos el vector a   a1 , a2 , a3  en el espacio tridimensional y los ángulos  ,  y  formados por los ejes de coordenadas positivos y el vector

a   a1 , a2 , a3  ; es decir:  

i , a  ,    j , a  ,    k , a  . Si a // L

(recta) donde a   a1 , a2 , a3  diremos que:

i) ii)

a1 , a2 , a3 son los números directores de la recta L. Los ángulos  ,  y  se llaman ángulos directores de la recta L, y son formados por los rayos positivos de los ejes coordenadas y la recta, respectivamente. Los ángulos directores toman valores entre 0o y 1800, es decir: 00   ,  ,   1800 iii) A los cosenos de los ángulos directores de la recta L, es decir: se denominan cosenos directores. 5.

EXPRESIONES DE LOS COSENOS DIRECTORES DE UNA RECTA DETERMINADOS POR DOS DE SUS PUNTOS Sea L una recta que pasa por los puntos p1  x1 , y1 , z1  y p2  x2 , y2 , z2  Si d  p1 , p2  || p1 p2 ||  ,  y 

son los

ángulos directores de la recta L, entonces se tiene:

cos  

x2  x1 , d  p1 , p2 

cos  

z2  z1 d  p1 , p2 

cos  

y2  y1 , d  p1 , p2 

4

6.

RELACIÓN ENTRE LOS COSENOS DIRECTORES DE UN RECTA TEOREMA La suma de los cuadrados de los cosenos directores de una recta L igual a 1, es decir: cos2   cos2   cos2   1 Aplicando la parte 5 se tiene: x x y y z z cos   2 1 , cos   2 1 , cos   2 1 , de donde d d d

 x2  x1    y2  y1    z2  z1  , por lo tanto 2 2 2 x2  x1   y2  y1   z2  z1   2 2 2 cos   cos   cos     2

d

2

2

d2

d2

d2

1

cos2   cos2   cos2   1 OBSERVACIÓN Si a   a1 , a2 , a3 

es

un

vector

dirección

de

la

recta

L,

donde:

|| a || a12  a22  a32 , entonces:



 i, a 



cos  

i.a a  1 || a || || a ||



a1 || a || cos 



 j, a 



cos  

j.a a  2 || a || || a ||



a2 || a || cos 



k, a



cos  

a k .a  3 || a || || a ||



a3 || a || cos 





a  || a || cos  ,|| a || cos  ,|| a || cos  || a ||  cos  ,cos  ,cos   LA RECTA 7.

LA RECTA EN EL ESPACIO TRIDIMENCIONAL Dado un punto p0  x0 , y0 , z0  y un vector a   a1 , a2 , a3  no nulo, llamaremos recta que pasa por p0  x0 , y0 , z0  paralela al vector a   a1 , a2 , a3  al conjunto.





L  p  R3 / p  p0  ta, t  R 8.

ECUACIÓN VECTORIAL DE LA RECTA Sea L la recta que pasa por el punto p0  x0 , y0 , z0  paralelo al vector a   a1 , a2 , a3  . Si p  x, y, z  de R3 es un punto cualquiera de la recta L, entonces el 5

vector p0 p es paralelo al vector a , es decir: p0 p // a   t  R tal que:

p0 p  t a , de donde entonces p  p0  ta , por lo tanto la recta L es dado por:





L  p  p0  ta / t  R

Ecuación vectorial de la recta L.

OBSERVACIONES.  Para cada par de puntos distintos de R3, hay una y solo una recta que pasa por ellos.  Consideramos la recta L  p0  ta / t  R . Un punto p de R3 pertenece a la





recta L si p  p0  ta para algún t en R, es decir:

p  L  p  p0  ta para algún t real 9.

ECUACIÓN PARAMÉTRICA DE LA RECTA EN EL ESPACIO Consideremos la ecuación vectorial de la recta L:





L  P0  ta / t  R De la observación anterior se tiene:

p  L  p  p0  ta para algún t real

De donde, al reemplazar las coordenadas de P, P0 y de las componentes del vector a se tiene:  x, y, z    x0 , y0 , z0   t  a1 , a2 , a3  , es decir:

 x  x0  a1t  L :  y  y0  a2t , t  R z  z  a t 0 3  Las cuales se conocen con el nombre de ecuaciones paramétricas de la recta L. OBSERVACIÓN Las ecuaciones paramétricas de la recta L que pasa por el par de puntos P1(x1,y1,z1) y P2(x2,y2,z2) esta dado por

 x  x1   x2  x1  t  L :  y  y1   y2  y1  t , t  R   z  z1   z2  z1  t

6

10.

ECUACIÓN SIMÉTRICA DE LA RECTA Consideremos las ecuaciones paramétricas de la recta L:  x  x1  a1t  L :  y  y1  a2t , t  R z  z  a t 1 3 

Suponiendo que a1  0, a2  0, a3  0 , despejando el parámetro t de cada x  x0 y  y0 z  z0 ecuación tenemos: t  , de donde por igualdad:   2 2 2 x  x0 y  y0 z  z0 L:   a1 a2 a3 Que se denomina simétrica de la recta L. OBSERVACIÓN 1. Si a3 =0, la ecuación simétrica de la recta L se describe en la forma x  x0 y  y0 L:   z  z0 a1 a2 2. Si a1   a3  0 . La ecuación simétrica de la recta L se escribe en la forma

L : x  x0  z  z0 11.

RECTAS PARALELAS Y ORTOGONALES Las relaciones de paralelismo y ortogonalidad entre dos rectas se dan comparando sus vectores direccionales Consideremos las ecuaciones vectoriales de dos rectas. L1  p0  ta / t  R L2  q0   b /   R Y









La recta L1 y la recta L2 son paralelas (L1 // L2) si y solo si, sus vectores direccionales son paralelos, es decir: L1 // L2  a // b

La Recta L1 y la recta L2 son ortogonales  L1  L2  si y solo si sus vectores sus vectores direccionales son ortogonales, es decir: L1  L2  a  b OBSERVACIOES 1. Si L1 y L2 son paralelas (L1 // L2), entonces L1 = L2 ó L1  L2  

7

2.

12.

Si L1 y L2 no son paralelas (L1 // L2), entonces L1  L2   (las rectas se cruzan) ó L1  L2 consta de un solo punto.

ÁNGULO ENTRE DOS RECTAS Consideremos las ecuaciones de dos rectas L1  p0  ta / t  R y L2  q0   b /   R









Un ángulo entre las rectas L1 y L2 se define como el ángulo formado por sus vectores direccionales a y b , es decir:

 L1 , L2  

 a, b    , y es dado por la formula

a.b , 0   || a || || b || DISTANCIA MÍNIMA ENTRE DOS RECTAS (RECTAS QUE SE CRUZAN) Si L1  p0  ta / t  R y L2  q0   b /   R son dos rectas no paralelas cos  

13.









(rectas que se cruzan), entonces a la distancia mínima entre L1 y L2 denotaremos por d(L1,L2) y es definido como el segmento perpendicular común entre ambas rectas. Si las rectas L1 y L2 se cruzan, quiere decir que existen planos paralelos que contienen a las rectas L1 y L2 respectivamente.

Si d es la distancia entre los planos P1 y P2 de donde N es normal al plano P2; por lo tanto N es ortogonal a los vectores a y b entre N  a x b . Ahora consideremos el vector unitario en la dirección de la normal N ; N N  y como    N , AC entonces || N ||



cos  

 N . AC ||  N || || AC ||



 N . AC || AC ||



, de donde  N . AC || AC || cos  ...(1)

Por otro lado en el triángulo rectángulo ABC se tiene: d || AC || cos 

…(2)

de donde al comprar (1) y (2) se tiene: d  L1 , L2  |  N . AC |

8

14.

TEOREMA Sean L1  p0  ta / t  R y L2  q0   b /   R dos rectas no paralelas (rectas









que se cruzan). La distancia mínima entre L1 y L2 esta dado por:

d  L1 , L2   15.



|| a x b ||

TEOREMA La distancia del punto P a la recta L1  p0  ta / t  R es dado por:



d  L1 , L2  

16.



| p0 q0 . a x b |





|| p0 p ||2 || a ||  p0 p.a



|| a || PROYECCION ORTOGONAL SOBRE UN PUNTO Consideremos una recta L1  p0  ta / t  R y un punto p, que no pertenece a la





recta L. Entonces la proyección ortogonal del punto p sobre la recta L es el punto A de la recta L, al cual denotaremos proyLP de tal manera que el vector AP sea ortogonal a la recta L. Observando el gráfico se tiene:

P0 A  proyaP0 P de donde A  P0  proyaP0 P A  P0  proyaP0 P , es decir:  A  proyLP  p0  proyaP0 P

9

PROBLEMAS DE RECTAS EN R3 1. Hallar la ecuación de la recta que pasa por el punto A(3,1,-2) y es perpendicular x 1 y  2 z 1 y corta a la recta L :   1 1 1 SOLUCIÓN Si L   1, 2, 1  t 1,1,1 / t  R

La recta pedida que pasa por A(3,1,-2) es: L1   3,1, 2    a, b, c  /   R Como L  L1  1,1,1 .  a, b, c   0  a  b  c  0

a  b  c  0

...(1)

Sea p  L  L1 entonces p  L  p  L1 de donde: Si p  L  p  1  t , 2  t , 1  t  , p  L1  p  3  a,1  b, 2  c  , entonces:  1  t , 2  t , 1  t    3   a,1  b, 2  c  de donde:

1  t  3   a  2  t  1  b 1  t  2   c  Entonces

5 ac 1  ba

 

c  5b  4a

...(2)

De (1) y (2) se tiene: a = 2b, c = -3b, (a,b,c)= (2b,b,-3b) = b(2,1,-3) Por lo tanto la recta pedida: L  3,1, 2     2,1, 3 /   R 2. Hallar la ecuación de la recta que pasa por el punto (3,-3,4) y es perpendicular a x  2 y 3 z  2 x 3 2y 7 3  z     cada una de las rectas L1 : y L2 : 2 1 5 1 2 3 SOLUCION Rectas L1 y L2 en su forma vectorial L1   2,3, 2   t  2, 1,5 / t  R y L2   3,7 / 2,3    1,1,3 /   R

como

L  L1   2, 1,5   a, b, c    2, 1,5  a, b, c   0 L  L2  1,1,3   a, b, c  

1,1,3 a, b, c   0

entonces

2a  b  5c  0 3a a  a 3a  a de donde c   , b  ,  a, b, c    a, ,   8,1,3  8 8  8 8  8 a  b  3c  0 Por lo tanto

 L  3, 3, 4   t 8,1, 3 / t  R

10

3. Hallar la ecuación de la recta que pasa por el punto M(-1,2,-3) es perpendicular x 1 y  1 z  3 al vector a   6, 2, 3 y se corta con la recta L1 :   3 2 5 SOLUCION Escribiendo la recta L1 en su forma vectorial: L1 1, 1,3  t  3, 2, 5 / t  R Sea p  L1  L  p  L1  P  L :

Si p  L1  p 1  3t , 1  2t ,3  5t  para algún t  R Como b  MP  P  M   3t  2, 2t  3, 5t  6  Además a  b  a.b  0   6, 2, 3 .  3t  2, 2t  3, 5t  6   0

6  3t  2   2  2t  3  3  5t  6   0 Por lo tanto: L  1, 2, 3  t  2, 3,6  / t  R

t  0, b   2, 3,6 

4. Dados los puntos A(3,1,1) y B(3,-2,4). Hallar el punto C de la recta L 1, 1,1  t 1,1,0  / t  R tal que  AB, AC  600





SOLUCIÓN Sea C  L  C 1  t.  1  t ,1 AB. AC || AB |||| AC || cos 600 , donde

AB   0, 3,3 , AC   t  2, t  2,0  || AB || 9  9  3 2 , || AC || 2  t  2   2 | t  2 | 2

Como AB. AC || AB |||| AC || cos 600 , reemplazando: 1 6  3t  3 2. 2 | t  2 | | t  2 | 2  t de donde t  2  0 como t  2 entonces 2 C 1  t , 1  t ,1 para t  2 5. Una recta pasa por el punto p(1,1,1) y es paralela al vector a  1, 2,3 , otra recta pasa por el punto Q(2,1,0) y es paralela al vector b   3,8,13 . Demostrar que las dos rectas se cortan y determinar su punto de intersección. SOLUCION Sean L1  1,1,1  t 1, 2,3 / t  R y L2   2,1,0    3,8,13 /   R Las recatas L1y L2 se cortan si y solo si P0 tal que P0  L1  L2 como P0  L1  L2  P0  L1  P0  L2 11

Si P0  L1  P0 1  t ,1  2t ,1  3t  P0  L2  P0  2  3 ,1  8 ,13  Como P0 es punto común a L1y L2 entonces: 1  t,1  2t,1  3t    2  3,1  8,13  1  t  2  3  1  2t  1  8 resolviendo se tiene t  4,   1 1  3t  13  Remplazando el punto de intersección es P0(5,9,13) 6. Dadas las rectas L1   3,1,0   t 1,0,1 / t  R

y

L2  1,1,1    2,1,0 /   R Hallar el punto Q que equidista de ambas rectas una distancia mínima, además hallar esta distancia SOLUCION Sea A  L1  A  3  t ,1, t  , B  L2

B 1  2 ,1   ,1 , AB  B  A   2  t  2,  ,1  t  a  AB  a. AB  0, 1,0,1 .  2  t  2, ,1  t 

de donde 2  2t 1  0 …(1) b  AB  b. AB  0   2,1,0  .  2  t  2, ,1  t   0 5  2t 1  0

…(2)

2  2t  1  0 formando el sistema de (1) y (2) se tiene  5  2t  4  0 1 resolviendo el sistema se tiene t  ,   1 2  A B   13 3 3  como Q es punto equidistante de A y B entonces Q    Q , ,   2   4 2 4 La distancia mínima d  Dadas las tres rectas

1 6 d  A, B   2 4

L1  1,1, 2   t 1, 2,0  / t  R

L2   2, 2,0    1, 1,1 /   R L3   0,3, 2   r  5,0, 2  / r  R

12

7. Hallar la ecuación de la recta que corte a estas tres rectas en M, N y P respectivamente de tal manera que MN  NP SOLUCION

M  L1  1,1, 2   t 1, 2,0  / t  R  M 1  t ,1  2t , 2 

N  L1   2, 2,0    1, 1,1 /   R  N  2  , 2  ,  

P  L1   0,3, 2   r  5,0, 2  / r  R  P  5r,3, 2  2r  Como MN  NP entonces se tiene donde    t  1,   2t  1,   2  = vectores se tiene

  t  1  5r    2    2t  1  1     2  2 r    2 

MN  N  M     t  1,   2t  1,   2  NP  P  N   5r    2,1   , 2r    2  ,

 5r    2,1  , 2r    2  ,



de

por igualdad de

5r  2  t  3 ...(1)  ...(2) 2  2t  0 2r  2  0 ...(3) 

de (2) y (3) se tiene   t , r   ahora reemplazamos en la ecuación (1). 3 3 3  1   7 1 3   15  t   ,   , r  , Luego M   , 2, 2  , N  , ,  , P  ,3, 1 2 2 2  2  2 2 2  2  Por lo tanto:

 1    L    , 2, 2   t 8,5, 1 / t  R    2 

8. Hallar la ecuación de la recta que pasa por p(19,0,0) y corta a las rectas L1   5,0, 1  t 1,1,1 / t  R , y L2   1, 2, 2   r  2,1,0  / r  R SOLUCION

13

Sean

A  L1   5,0, 1  t 1,1,1 / t  R  A  t  5, t , t  1

B  L2   1, 2, 2   r  2,1,0  / r  R  B  2r  1, r  2, 2  como los puntos P, A, B son colineales, entonces: PA // AB   m  R tal que PA  mAB de donde A  P  m  B  A  que al reemplazar por sus coordenadas se tiene:

 t  14, t , t  1  m  2r  t  6, r  t  2, t  3 t  14  2mr  mt  6m ...(1)  por igualdad de vectores se tiene: t  rm  mt  2r ...(2) t  1   mt  3m ...(3)  3m  1 m 1 ,r  de la ecuación(1) m 1 m 1  m  t  2mr  6m  14 reemplazando t y r se tiene: de la ecuación (3) y (2) se tiene: t 

m

15 28 4 , t , r 11 13 15

luego a  PA   t  14, t , t  1 para t 

28  145 28 15  , a   , ,  13  13 13 3 

 L  19.0.0   t  154, 28,15  / t  R 9. Encuentre

el

punto

de

intersección de x7 y z L1   1,7,17   t  1, 2,3 / t  R y L2 :   4 1 5

las

rectas:

SOLUCION Escribiendo la ecuación L 2 en forma vectorial. L2   7, 0, 0     4,1,5  /   R

Sea p  L1  L 2 entonces p  L1  p  L 2 .

Si p  L1  p  1  t , 7  2t ,17  3t   p  L 2  p  7  4 ,  ,5  como p  L1  L 2   1  t , 7  2t ,17  3t    7  4 ,  , 5  1  t  7  4  entonces t  4,   1 Luego: p  3, 1,5  7  2t   17  3t  5  10. Hallar la ecuación vectorial de la recta que intercepta en ángulo recto a las rectas L1   3,3, 4   t  2, 2,3 / t  R , L2  1,6, 1    1, 2,0  /   R SOLUCION

14

Sean A  L1  A  3  2t ,3  2t , 4  3t  B  L2  B 1   , 6  2 , 1 como A, B son puntos sobre la recta L entonces el vector dirección de la recta L es a  AB  B  A de donde se tiene:

a   2  2t   ,3  2  2t , 5  3t  como L  L1 , L2 entonces: a.  2, 2,3  0  a.  1, 2, 0   0

17t  2  132  resolviendo el sistema se tiene t = -1,  = -2 2t  5  8

por lo tanto los puntos son A(1,1,1), B(3,2,-1), a  AB  B  A   2, 1.2  , entonces la recta pedida es:

L  1,1,1  t  2, 1, 2  / t  R 11. Encontrar la ecuación de la recta que pasa por el punto p(7,-2,9) y es perpendicular a las rectas SOLUCION: Los vectores direcciones de L1 y L2 a  (2, 2,3), b  (2,5, 2) respectivamente.





Sea L la recta que pasa por el punto p(7,-2,9), luego la recta pedida L= (7,-2,9)+t b / t  R pero como L  L1 , L 2 en tonces c  a, b entonces: i j k c  a  b  2 -2 3  (11,10,14) 2 5 -2 Por lo tanto: L= (7,-2,9)+t(11,10,14) / t  R 12. Hallar la ecuación vectorial de la recta que intercepta en ángulo recto a las rectas L1 = (3,3,4)+t(2, 2,3) / t  R , L2 = (1,6,-1)+ (1, 2,0) /   R SOLUCION: Sean A  L1  A(3  2t ,3  2t , 4  3t )

B  L2  B(1   , 6  3  2 , 1)

Como A,B son puntos sobre la recta L entonces el vector dirección de la recta L es

15

a  AB  B  A de donde se tiene: a  (2  2t   ,3  2  2t , 5  3t ) como L  L1 , L 2 entonces: a.(2, 2,3)  0 17t  2  13  resolviendo el sistemas se tiene t= -1,   2,  a(1, 2, 0)  0 2t  5  8 por lo tanto los puntos son A(1,1,1), B(3,2,-1), a  AB  B  A  (2, 1, 2). Luego la ecuacion vectorial de la recta pedida es:

L= (1,1,1)+t(2, 1, 2) / t  R 13. Determinar una recta tal que con las rectas L1  (2,1, 4)  t (1,1,0) / t  R y L1  (2   ,1   ,3   ) /   R

determinan un

triangulo de area 5u2. SOLUCION:

Sea p  L1  L2  p  L1  p  L1 Si p  L1  p(2  t ,1  t , 4) p  L2  p (2   ,1   ,3   ) como p  L1  L2 , entonces: (2  t ,1  t , 4)  (2   ,1   ,3   ) 2  t  2    de donde: 1  t  1   al resolver el sistema se tiene que: t=  1 4  3    por lo tanto el punto p es p(3,2,4), ahora tenemos en t cercano a p asi como t=2 entonces el punto A de L2 es A(4,3,4), ademas B  L1  B(2   ,1   ,3   ) entonces se tiene: a  AB  B  A  (  2,   2,   1) por otra parte b  AP  P  A  (1, 1, 0) 1 ademas el area A= a  b  5 de donde a  b =10 entonces  2  2  49  0 de 2 donde se tiene: 1  1  5 2,  2  1  5 2 por lo tanto las rectas pedidas son:

 L= (4,3,4)+t(-1  5



L= (4,3,4)+t(-1  5 2, -1  5 2,5 2) / t  R



2, -1  5 2, 5 2) / t  R

14.Sea A(1,1,2) un punto y supongamos que la recta L tiene por ecuaciones paramétricas a: x=4-t, y y=5+3t,z=3+t, t  R , encontrar un punto B en L, tal que A-B y la recta sean perpendicular.

16

SOLUCION:

Sea L= (4,5,3)+t(-1,3,1) / t  R b  P0 A  A  P0  (3, 4, 1) a.b .a || a ||2 (1,3,1).(3, 4, 1) P0 B  .(1,3,1) 11 10 10 30 10 P0 B   (1,3,1)  ( ,  ,  ) 11 11 11 11 10 30 10 10 30 10 P0 B  B  P0  ( ,  ,  )  B(4  ,5  ,3  ) 11 11 11 11 11 11 10 30 10  B( ,  ,  ) 11 11 11 P0 B  proyab 

15. Determinar los ángulos entre una recta L paralela al vector y los ejes coordenadas. SOLUCION:

Sea L  P0  ta / t  R , donde a  (1,1,1) es la dirección de la recta L y ||a || 3 entonces: a 1 cos   1  || a || 3

a2  || a || a cos   3  || a || cos  

1 3 1 3

 



  arc cos(

1 ) 3

1 ) 3 1   arc cos( ) 3

  arc cos(

16. Hallar la longitud del menor segmento horizontal (paralelo al plano XY) que une las rectas SOLUCION: L1 = (1,2,0)  t(1, 2,1) / t  R L2 = (0,0,0)   (1,1,1) /   R Si A  L1  A(1  t , 2  2t , t ), B  L 2  B( ,  ,  )

17

como AB // al plano XY entonces   t Luego A(1  t , 2  2t , t ) y B(t , t , t ) d || AB || 1   t  2   0 de donde f (t )  t 2  4t  5 2

f '(t ) 

t2 t 2  4t  5

 0  t  2 número critico  d || AB || 1  0  0  1  d  1

17. Dadas las rectas .Hallar la ecuación de la perpendicular comun. SOLUCION: Las rectas L1 y L2 no son paralelas, es decir L1// L2. Ahora veremos si  p  L1  L2  p  L1  p  L2

Si p  L1  p(1  2t , 2  3t ,5  4t ), p  L2  p( 2,1   , 2  2 ) (1  2t , 2  3t ,5  4t )  (2,1   , 2  2 ) de donde

 3 t  2 1  2 t   2   15   2  3t  1       2 5  4t  2  2   13    2  por lo tanto las rectas L1 y L2 son rectas que se cruzan i j k a  2 3 4  10i  4 j  2k 0 1 2 L= (1, - 2,5)  t(10, 4, 2) / t  R ; L'= (-2,1,-2)   (10, 4, 2) /   R 18. Determinar bajo que dirección debe ser lanzada rectilíneamente una partícula desde el punto A(2,2,3), hacia la recta L  (0,1   ,  ) /   R para que lo alcance al cabo de dos segundos, siendo su velocidad V  3u / seg

Sea B  L  B(0,1   ,  ) para algún   R además e  vt donde e  d ( A, B) para t  2seg. V  3u, e  2 3 d ( A, B)  4  (  1) 2  (  3) 2  2 3 de donde  2  2  1  0    1 Luego B(0,0,1) entonces está dado por el vector AB  B  A  (2, 2, 2)

 AB  (2, 2, 2) 18

19. Determinara la ecuación de la recta que pasa por el punto medio de AB y corta bajo un ángulo de 60º a la recta que pasa por los puntos R y S, donde A(2,4,0), B(0,0,-2), R(3,3,3), S(-1,3,3). SOLUCION El punto medio del segmento

AB es M(1,2,-1), y

observando el grafico este problema tiene dos soluciones. La ecuación de la recta L1 que pasa por R y S es:

L1 (1,3,3)  t (0,0,0) / t  R Sea N el Punto de intersección de L con L1 es decir:

Si N  L1  N ( 1  t ,3,3) pasa algún t  R. Definimos b  MN  N  M  (t  2,1, 4), como 60º  ( L, L1 )  (a, b) entonces : cos 60 

a.b

; donde a  (1, 0, 0) y b  (t  2,1, 4)

a.b cos60° 

(1, 0, 0).(t  2,1, 4) (t  2) 2  1  16



1 (t  2)  2 (t  2) 2  17

(t  2) 2  17  4(t  2) 2  t  2 

17 17  b  ( ,1, 4) 3 3

por lo tanto las soluciones son: 17 17     L  (1, 2, 1)   ( ,1, 4) /   R  ; L '  (1, 2, 1)  r (  ,1, 4) / r  R  3 3    

20. Dados los vértices del triángulo A(3,-1,-1), B(1,2,-7) y C(-5,14,-3). Hallar las ecuaciones simétricas de la bisectriz del ángulo interno del vértice B. SOLUCION Tomemos los vectores unitarios u y v en las direcciones de

BA y BC y respectivamente donde:

BA  (2, 3, 6), BC  (6,12, 4)

1 (2, 3, 6) BA 1 BC 1 u  (2, 3, 6) y v   (3, 6, 2) 7 || BA || 7 || BC || 7 entonces sea b  u  v el vector de la dirección de la directriz BD es decir: 19

1 1 b  (1,3,8)   (1, 3, 8). Luego los números directores de la bisectriz BD son 7 7

1, 3, 8. Si B(1,2,-7) pertenece a la bisectriz, entonces sus ecuaciones simétricas son: L:

x 1 y  2 z  7   1 3 8

EL PLANO DEFINICIÓN.Un plano es un conjunto P de puntos p(x,y,z) de R3. Si existe un punto p0(x0,y0,z0) de R3 a  (a1 , a2 , a3 ) y b  (b1 , b2 , b3 ) de R3 de tal manera y dos vectores no paralelos que:





P  P( x, y, z )  R3 / P( x, y, z )  P0 ( x0 , y0 , z0 )  ta  b, t ,   R ECUACIÓN VECTORIAL DEL PLANO.-

Consideremos un plano P que pasa por el punto p0(x0,y0,z0) y que es paralelo a los vectores paralelos a  (a1 , a2 , a3 ) y b  ( b1, b2, b3) . p  P entonces existen t ,   R tal que:

Sea

p0 p  ta   b ,de donde p  p0  ta   b entonces: p  p0  ta   b ,



luego



P  p0  ta   b / t ,   R Que es la ecuación vectorial del plano P. OBSERVACION.-

1. De la ecuación vectorial del plano





P  p0  ta   b / t ,   R

se obtiene la normal del plano que es una recta perpendicular a dicho plano: N  a  b .

20

P   p0  ta   b / t ,   R 2. Si N es una normal al plano y si p1 , p2  P entonces N es ortogonal a p1 p2  p2  p1 .

P   p0  ta   b / t ,   R 3. Si N es la normal al plano y si p2  p1 es ortogonal a N entonces p  P .

4. Si p0 es un punto fijo del plano P y N es su normal, entonces la ecuación del plano es

P : N .( p  p0 )  0

Es la ecuación del plano que pasa por p0 y cuya normal es N .

ECUACIONES PARAMÉTRICAS DEL PLANO.Consideremos el plano.





P  p0  ta   b / t ,   R

p  p0  ta   b Si p  P entonces para t ,   R , reemplazando por sus respectivas (x,y,z) = (x 0 ,y0 ,z0 )+t(a1 , a2 , a3 )   (b1 , b2 , b3 ) componentes se tiene: de donde por igualdad se tiene:

21

 x  x0  a1t  b1  P:  y  y0  a2t  b2  t, ,  R z  z  a t  b  0 3 3  Que son las ecuaciones paramétricas del plano P. ECUACION GENERAL DEL PLANO.Sea P el plano que pasa cuyo vector normal es:

por el punto p0 (x 0 ,y0 ,z0 )

N =(A,B,C). Si p  P entonces: p0 p  N , de donde p0 p.N  0 N .( p  p0 )  0 . entonces: Ahora reemplazando por sus componentes: (A,B,C).(x-x0,y-y0,z-z0) = 0 entonces A(x-x0) + B(y-y0) + C(z-z0) = 0 Ax + By + Cz + (-Ax0-By0-Cz0) = 0, de donde P: Ax + By +Cz + D = 0. Que es la ecuación general P. PLANOS PARALELOS Y ORTOGONALES.Consideremos los planos: P1: A1x + B1y + C1z + D1 = 0 y P2: A2x + B2y + C2z + D2 = 0, donde N  ( A1 , B1 , C1 ) y N 2  ( A2 , B2 , C2 ) son sus normales, respectivamente, entonces: i)El plano P1 es paralelo al plano P2 (P1// P2) si y solo si sus normales N 1 y N 2 son paralelas, es decir:

P1 // P2  N 1 // N 2

Si,

22

N1 // N 2  r  R tal que N1  N 2 , lo que quiere decir que los coeficientes de las ecuaciones cartesianas de los planos deben ser proporcionales, o sea que debe cumplirse:

A1 B1 C1   r A2 B2 C2 Si los planos P1 y P2 son paralelos puede ocurrir que: P1  P2 ó P1  P2   es decir:

P1 // P2  P1  P2 ó P1  P2   ii) El plano P es ortogonal al plano P2 P1  P2  si y solo si sus normales N1 y N 2 son ortogonales, es decir:

P1  P2  N1  N 2

Si N1  N 2  N1.N 2  0  A1 A2  B1B2  C1C2  0 , por lo tanto P1  P2  A1 A2  B1B2  C1C2  0

INTERSECCIÓN DE PLANOS.Consideremos los planos: P1: A1x + B1y + C1z + D1 = 0 y P2: A2x + B2y + C2z + D2 = 0. Si el plano P1 no es paralelo al plano P2 (P1 // P2) entonces la intersección de P1 y P2 nos da una recta L. es decir: Si P1 // P2   L tal que P1  P2  L

23

ECUACIÓN BIPLANAR DE LA RECTA.A la ecuación de una recta que es la intersección de dos plano se denomina ecuación biplanar de la recta y se expresa en la forma siguiente:

A x  B1 y  C1 z  D1  0 L: 1 A 2 x  B 2 y  C 2 z  D 2  0 La ecuación biplanar de la recta se expresa en forma vectorial, paramétrica y simétrica. El vector dirección a de la recta se determina en la forma siguiente:

a= N 1  N 2 , donde N 1 y N 2 son las normales de los planos P 1 y P2 respectivamente: i j k a = N 1  N 2  A1 B1 C1  (0, 0, 0) A2 B2 C2 p (x y , z ) El punto 0 0, 0 0 por donde pasa la recta se determina resolviendo el sistema de ecuaciones de los planos P1 y P2.

INTERSECCIÓN ENTRE RECTA Y PLANO.Consideremos la ecuación general de un plano: P: Ax + By + Cz + D = 0 y la ecuación vectorial de la recta L  p0  ta / t  R si L y P no son paralelos entonces al intercectarse nos da un punto Q, es decir:

L  P  Q Para calcular el punto Q de intersección se resuelve el sistema de ecuaciones de la recta L y el plano P. PLANO PARALELO A UNA RECTA Y PLANO PERPENDICULAR A UNA RECTA.Consideremos la ecuación general del plano P: Ax + By + Cz + D = 0. donde N = (A,B,C) es la normal y la ecuación vectorial de la recta L  p0  t a / t  R donde a es el vector dirección. 24

La recta L es paralela al plano P si solo si el vector dirección a es ortogonal al vector normal N es decir: L // P  a  N Si la recta L es paralela al plano P puede ocurrir que la recta L está contenida en el plano P ó que la intersección es el  , es decir: Si L//P  L  P ó L  P  

La recta L es perpendicular al plano P si y solo si el vector dirección a de L paralelo al vector normal N de P, es decir: L  P  a // N

FAMILIA DE PLANOS.En forma similar que en la geometría analítica plana, en donde se consideraba una familia de rectas, en este caso se puede considerar una familia de planos, por ejemplo, la ecuación 2x - y + 3z + D = 0 representa una familia de planos paralelos donde su normal es N = (2,-1,3). Una familia de planos importante, es el sistema de planos que pasan por la intersección de dos planos dados, cuyas ecuaciones se expresan: P1: A1x + B1y + C1z + D1 = 0

….(1)

P2: A2x + B2y + C2z + D2 = 0 Los puntos p(x,y,z) que satisfacen a la ecuación (1) están sobre la recta de intersección, dichos puntos p(x,y,z) también satisfacen a la ecuación: K1(A1x + B1y + C1z + D1) + K2(A2x + B2y + C2z + D2) = 0

….(2)

donde K1 y K2 son números reales cualesquiera excepto que sean ceros simultáneamente. Si en la ecuación (2) se tiene que K1  0 , entonces a la ecuación (2) se puede expresar en la forma: A1x + B1y + C1z + D1 + K2(A2x + B2y + C2z + D2) = 0

….(3)

A la ecuación (3) se denomina la familia ce planos que pasan pors la intersección de los planos P1 y P2 25

DISTANCIA DE UN PUNTO A UN PLANO.Consideremos la ecuación general de un plano P: Ax + By +Cz + D = 0 y un punto p1(x1,y1,z1) que no pertenece al plano P.

Consideremos un vector unitario  N en la dirección del vector normal, es decir

N 

N



N

1 A  B2  C 2 2

( A, B, C )

como  = ( p0 p1 ,  N ) entonces p0 p1. N  p0 p1 cos  En el triangulo rectangulo se tiene: d ( p1, P)  p0 p1 cos  de (1) y (2) se tiene que: 1

d ( p1, P )  p0 p1. N  

A2  B 2  C 2

( A, B, C ).( x1  x0 , y1  y0 , z1  z0 )

A( x1  x0 )  B( y1  y0 )  C ( z1  z0 ) A  B C 2

2

2

 d ( p1, P) 



Ax1  By1  Cz1  ( Ax0  By0  Cz0 ) A2  B 2  C 2

Ax1  By1  Cz1  D A2  B 2  C 2

OBSERVACION.- Dadas las ecuaciones generales de dos planos paralelos P1: Ax + By + Cz + D1 = 0 y P2: Ax + By + Cz + D2 = 0 La distancia entre dichos planos esta dado por la formula.

d ( P1, P2 ) 

D1  D2 A  B2  C 2 2

26

ANGULO ENTRE RECTAS Y PLANO.Consideremos la ecuación vectorial de una recta





L  p0  ta / t  R

y la ecuación general del plano P: Ax + By + Cz + D = 0 cuyo vector normal es N  ( A, B, C )

Sea   (a, N ) angulo entre los vectores a y N . entonces: cos  

a.N a N

, ademas tiene  =

 2

  , entonces:

 a.N a.N sen =sen(   )  cos   por lo tanto: sen = 2 a N a N Que es la expresion para calcular el angulo  formado por una recta y un plano PROYECION ORTOGONAL DE UN PUNTO SOBRE UN PLANO.La proyección ortogonal de un punto p sobre el plano P: Ax + By +Cz + D = 0 con p normal N  ( A, B, C ) es el punto p del plano P, al cual denotaremos por Pr oyP , de tal 0

manera que el vector p0 p1 es ortogonal al plano P. Para hallar el punto p0 trazamos por L  p tN /t R el punto p una recta L ortogonal al plano P es decir: de donde L  P  p0





27

PROYECCION ORTOGONAL DE UNA RECTA SOBRE UN PLANO.La proyección ortogonal de la recta



 sobre el plano P: Ax + By + Cz

L  p0  ta / t  R

L

+ D = 0, es la recta L , el cual denotaremos por Pr oyP que esta contenida en el plano P y que pasa por dos puntos de P que son las proyecciones ortogonales de dos puntos de L sobre el plano P.

DISTANCIA MINIMA ENTRE UN PLANO Y UNA RECTA QUE NO ESTA CONTENIDA EN EL PLANO.La distancia mínima entre una recta L  p0  ta / t  R N  ( p  Q0 )  0 , y un plano donde la recta L no esta contenida en el plano P y además L es paralela a P es dado por la formula.





d ( L, P)  compN Q0 p0 

Q0 p0 .N N

ANGULO ENTRE DOS PLANOS.Consideremos las ecuaciones generales de dos planos P1: A1x + B1y + C1z + D1 = 0, cuya normal es N  ( A1 , B1 , C1 ) y P : A2 x  B2 y  C2 z  D2  0 cuya normal es N 2  ( A2 , B2 , C2 ) . El angula ө formado por los planos P1 y P2 es igual al ángulo entre sus vectores normales N 1 y N 2 respectivamente y es dado por la expresión siguiente:

cos =

N 1 .N 2 N1 N 2

28

PROBLEMAS DE PLANO EN R3 1. Hallar la ecuación vectorial de la recta L, dado por la intersección de los planos P1: 3x + y – 2z = 5 ; P2: x + 2y + z + 5 = 0. SOLUCIÓN: Calculando el vector dirección a de la recta L.

i j k a  3 1 -2  (5, 5,5)  5(1, 1,1) 1 2 1 ahora calculamos un punto de la recta L, para esto resolvemos el sistema de ecuaciones.

3x + y - 2z = 5 5x +5 y = -5 entonces  , simplificando  x  2 y  z  5  0 x  y  0 ahora damos un valor a cualquiera de las variables de x e y por ejemplo para x = 0, y = 1,

z = -3 entonces p0(0,-1,3).

Luego la ecuación de la recta L en forma vectoriales:

L  (0, 1, 3)  t (1, 1,1) / t  R

2. Hallar la ecuación del plano que pasa por la intersección de los planos 2x – y – z + 8 = 0, x + 6y – 2z – 7 =0 y por el punto (1,-2,2) SOLUCIÓN: Aplicando el concepto de familia de planos se tiene: P: 2x – y – z + 8 +k( x + 6y – 2z – 7) =0

 P  2  2  2  8  k (1  12  4  7)  0  k como (1,-2,2)

5 11

P: 2x – y – z + 8 +5/11 ( x + 6y – 2z – 7) =0

 P : 27 x  19 y  21z  53  0

29

3.

Hallar el punto de intersección de la recta L :

x2 y z4 y el plano   3 1 2

P : 2 x  3 y  z  11  0 SOLUCIÓN: Escribiendo la recta L en forma vectorial. L  (2,0, 4)  t (3, 1, 2) / t  R

como L//P  p tal que p  L  P. Si p  L  P entonces p  L  p  P como p  P entonces p(-2+3t,-t,4+2t) para algun t  R. ademas p  P  2(2  3t )  3(t )  (4  2t )  11  0  t  3 Luego: p(-11,3,-2) 4.

Demostrar que la recta L  (2,1, 5)  t (3, 4, 4) / t  R es paralelo al plano

P : 4x  3 y  6z  5  0

SOLUCIÓN: Para demostrar que la recta L es paralela al plano P debe cumplirse que el vector dirección a de la recta es perpendicular al vector normal N del plano. Es decir: Luego como a.N= 0 entonces a  N. Por lo tanto la recta L es paralela al plano P.

5. Encontrar una ecuación del plano que pasa por los puntos de A(1,0,-1) y B(2,1,3) y





que además es perpendicular al plano P1  ( x, y, z )  R3 / x  y  z  2  0 SOLUCIÓN:

30

comP1  P2  N 1 // P, ademas se tiene que : A, B  P  AB  (1,1,4) como N  AB, N 1 entonces

i j k N  1 1 4  5(1, 1, 0) 1 1 -1 de donde tenemos que: N=  5(1, 1,0)

Luego P: N.(( x, y, z )  ( x0 , y0 , z0 ))  0 de donde  P: x-y=1 6. Hallar la ecuación del plano que pasa por la intersección de los planos 2x –y +3z = 2 y 4x+ 3y – z = 1y es perpendicular al plano 3x – 4y – 2z = 9 SOLUCIÓN: Sea Pα la familia de planos que pasan por la intersección de los planos 2x –y +3z = 2 y 4x+ 3y – z = 1 Pα: 2x –y +3z – 2 + α(4x + 3y – z -1) = 0 Pα: (4α + 2)x + (3α – 1)y + (3 – α)z – 2 – α = 0, donde su normal es: N  (4  2.3  1.3   ) y sea P: 3x-4y-2z=9 cuya normal es: N  (3, 4, 2) como P  P  N  N  N .N  0

(3,-4,-2).(4α + 2,3α – 1.3α) = =, de donde 12α + 6 - 12α + 4 – 6 + 2α = 0 entonces α = -2

 P : 6 x  7 y - 5z  0 7. Hallar el ángulo θ que forma la recta con el plano SOLUCIÓN: Sea  =(L,P) donde a  (1,1, 2) vector direccion de la recta N  (2, 1,1)

el

vector

normal del plano P. Ahora aplicamos la relación para calcular el ángulo θ.

31

sen =

a.N



a N

(1,1, 2).(2, 1,1) 2  1  2 1   6 2 6 6

de donde: sen =

1 entonces  =60° 2

8. Hallar la ecuación del plano que pasa por el punto p0(3,1,-2) y hace ángulos iguales con las rectas

L1  (1, 4, 2)  t (1,1,1) / t  R L2 : ejeOX , L3 : ejeOY

SOLUCIÒN: El plano pedido es: P: N .( p  p0 )  0 , de donde N  ( A, B, C ) y p0(3,1,-2) el punto por donde pasa el plano. La condición del problema es:

( L1 , P)  ( L2 , P)  ( L3 , P), donde para ( L1, P)  ( L2 , P), se tiene:

sen =

N .a



N a

N .b

, donde a  (1,1,1), b  (1, 0, 0), N  ( A, B, C )

N b

efectuando operaciones se tiene que: ( 3  1) A  B  C  0 para ( L2 , P)  ( L3 , P), se tiene: sen =

N .b N b



N .c

.....(1)

, donde b  (1, 0, 0), c  (0,1,1), N  ( A, B, C )

N c

efectuando operaciones se tiene: A=B

.....(2)

ahora reemplazamos (2) en (1) se tiene: C  ( 3  2) B como N  ( A, B, C )  ( B, B,( 3  2) B)  B(1,1, 3  2) B  0

Por lo tanto P: (1,1, 3  2) .(x-3,y-1,z+2) = 0  P:x+y+( 3  2) z  2 3  8  0

32

9. Sea   (a, b, c) y N  ( A, B, C ) vectores no nulos de R3 tal que N   si p0(x0, y0, z0) es un punto del plano π = Ax + By + Cz + D = 0. Demostrar que



 esta contenida en π.

L  p0  t  / t  R

SOLUCIÓN:

como N    N .  0  Aa  Bb  Cc  0 ademas





L  p0  t  / t  R  ( x0 , y0 , z0 )  t (a, b, c) / t  R por demostrar que L   : Ax  By  Cz  d  0 Sea p  L  p(x0  ta, y0  tb, z0  tc) como p0    A(x0  ta )  B( y0  tb)  C ( z0  tc )  D  0  Ax0  By0  Cz0  D  t ( Aa  Bb  Cc)  0 0

= 0 + t( Aa  Bb  Cc)  0  t0  0, entonces p   luego L   10. Hallar la ecuación del plano que pasa por el punto A83,4,1) y es ortogonal a los planos P1: x-y = 4, P2: x+y = 6 SOLUCIÓN:

Sea P1 : x  y  4 de donde N 1  (1, 1, 0) N 2  (1, 1, 0) : P2 : x  y  6 de donde N 2  (1, 0,1) P: N .( p  A)  0 es el plano pedido como P  P1 , P2 entonces N 1 , N 2 // P de donde la normal N de P es: i j k N  N 1  N 2  1 -1 0  (1, 1,1) 1 0 1 como P: N .( p  A)  0 , al reemplazar se tiene. P: (-1,-1,1).(x-3,y-4,z-1) = 0 P : x  y  z  6

33

11. Si P es un plano tal que:

P  eje x = (a,0,0)/a  0,a  R , P  eje y = (0,b,0)/b  0,a  R

. Demostrar que P

x y z P:    1 tiene la ecuación: a b c

SOLUCION:

Sea a  AB  B  A  (a, b, 0) b = AC = C - A  (a, 0, c) i j k N = a  b = -a b 0 =(bc,ac,ab) -a 0 c

La ecuacion del plano es: P: N.(p - A) = 0, reemplazando se tiene: P: (bc,ac,ab).(x-a,y,z) = 0  P: bcx +acy + abz = abc  P:

x y z   1 a b c

12. Si A,B,C y D son todos no nulos. Demuéstrese que el tetraedro formado por los planos coordenados y el plano P: Ax + By + Cz + D = 0 tiene un volumen igual a

1 D3 V 6 ABC SOLUCION: Sean P, Q, R, los puntos de intersección del plano P: Ax + By + Cz + D = 0, con los ejes

P(  coordinados respectivamente, es decir:

D D D ,0,0), Q(0,- ,0) y R(0, 0,  ) A B C

El volumen del tetraedro OPQRS es:

1 OPOQOR  de donde se tiene:  6 D D D OP = (  ,0,0), OQ = (0,- ,0) y OR  (0, 0,  ) A B C

V

 1 V 6

D 0 0 A D 1  D3 1 D3 0 =  B 6 ABC 6 ABC D 0 0 C

1 D3 V  6 ABC

34

13. Un plano pasa por el punto A(3,1,-1), es perpendicular al plano 2x – 2y + z = -4, y un intercepto Z es igual a -3, hallase su ecuación.

SOLUCION: Sea P1: 2x – 2y + z = -4, de donde N1  (2, 2,1) y P el plano por calcular. Luego como P1  P  N1 // P y como intercepto Z con P es -3 entonces B(0,0,3) es un punto del

plano P y además A, B  P  AB // P de donde AB  (3, 1, 2) como N1 , AB // P entonces la normal es N dado por

i jk N  N1  AB 2 -2 3  (11,10,14) 2 5 -2 P : N.( x  3, y  1, z  1)  0, de donde P: (5, 1,8).( x  3, y 1, z  1)  0, por lo tanto: P: 5x + y - 8z - 24 = 0

14. Hallar la ecuación de cada uno de los planos que se hallan a dos unidades del origen y tiene una normal que hace ángulos de 60° con los semi ejes positivos OX y OY.

SOLUCION:

Sea P el plano buscado, cuya normal es

N   cos  ,cos  ,cos  

como  = =60°  cos 2  cos 2   cos 2  1  cos = 

2 2

1 1 2 1 N  ( , , )  (1,1,  2) 2 2 2 2

La ecuación del plano es: P: x + y  2z + D = 0

como d (0, P)  2 

000 D 1  1  2

=2 de donde D =4  D = 4  D = -4

Si D = 4 entonces P1: x + y  2z + 4 = 0 D =-4 entonces P2 : x + y  2z - 4 = 0

35

15. Hallar la ecuación del plano perpendicular al plano z = 2, que contenga al punto (2,2,2) y que haga un ángulo de 60° con el plano

3x  2 y  3z  2  0

SOLUCION:

La ecuación del plano pedido es d la forma P: Ax + By + D = 0 puesto que es perpendicular al plano z = 2 paralelo al plano XY. La normal del plano P es:

N  ( A, B, O) Si P1 : 3 x  2 y  3 z  2  0, de donde N1  ( 3, 2, 3) El angulo formado por P1 y P es  =60° que es dado por: cos = cos 60 

3 A  2B 4 A2  B 2

de donde

N1.N N1 N

1 3 A  2B   2 A2  B 2  3 A  2 B 2 4 A2  B 2

4( A2  B 2 )  3 A2  4 B 2  4 3 AB  A  4 3B como (2,2,2)  P  2A+2B+D = 0

...(1) ...(2)

de (1) y (2) se tiene D =  (8 3  2) B reemplazando (1) y (3) en P: Ax + By + D = 0

...(3)

P: 4 3Bx + By - (8 3  2) B  0, B  0  P : 4 3x + y - 8 3  2  0 16. La recta L1  (5  t , t ,0) / t  R se refleja en el plano  : 2x  y  z . Hallar la ecuación de la recta reflejada

SOLUCION

Se observa que p 2  L1    p 2  L1  p 2   Si p 2  L1  p 2 (5  t , t , 0) para algun t  R además p 2   : 2(5  t )  t  0  1  0  t  3 de donde p 2 (2,3, 0) también p1 (5, 0, 0)  L1 como  : 2 x  y  z  1  0, de donde N  (2, 1,1) entonces N    N // L3

L3  (5, 0, 0)   (2, 1,1) /   R A  L3    A  L3  A  

36

A  L3  A  5  2 ,  ,   para algún   R, además A   entonces 3 2(5  2 )      1  0 entonces    , de donde: 2 3 3 3 3 3 3 A(2, ,  )  AP1  (2, ,  )  Bp1  p1  B  2 Ap1  2(3,  , )  (6, 3,3) 2 2 2 2 2 2 p1 p2  p1  p2  (3,3, 0)  Bp2  p2  B  (3, 0,3) como Bp2 // L y p2  L

entonces L  (2,3,0)  r (3,0,3) / r  R

17. Dado el plano

P : x  2 y  3z  8 y la recta L :

x  4 5 z  , y  1. Hallar la 4 3

ecuación de la recta que pasa por el punto (0, 2, -1) paralela al plano dado y corta la recta L. A la ecuación de la recta L :

x  4 5 z  , y  1, escribiremos en forma vectorial 4 3

L  (4, 4,5)  t (4,0,3) / t  R. Sea L1 la recta por determinara, es decir: L1  (0, 2, 1)  r (a, b, c) / r  R como

L1 corta a L   p  L1  L  p  L1  p  L Si p  L1  p(ra, 2  rb, 1  rc)  p  L  p(4  4t , 1,5  3t ) de donde por igualdad (ra, 2  rb, 1  rc)  (4  4t , 1,5  3t ) entonces.

4t  4  a  r 4  4t  ra   3   1  2  rb   b   r 5  3b  1  rc   6  3t  c  r 

...(1)

como P : x  2 y  3z  8 de donde N  (1, 2,3) como L1 // P entonces a  N donde a  (a, b, c) Si a  N  a.N  0  0 a  2b  3c  0 4t - 4 6 18  9t reemplazando (1) en (2) se tiene.   0t 4 r r r 12 3 6 3 de donde: a  , b   , c   como a  (a, b, c)  (4, 1, 2) r r r r

...(2)

L1  (0, 2, 1)   (4, 1, 2) /   R

37

18. El intercepto Y de un plano es menor en una unidad que su intercepto Z y mayor en dos unidades que su intercepto X, si el volumen encerrado por el plano y los tres planos coordenados es 15u3, hallar la ecuación del plano. SOLUCION: Los puntos por donde pasa el plano π son: (0,0,a), (0,a-1,0),(a-3,0,0) y la ecuación del plano es:

 : N .( x, y, z )  d donde N  ( A, B, C ) (0, 0, a)    ( A, B, C ).(0, 0, a)  d  aC  d (0, a  1, 0)    ( A, B, C ).(0, a  1, 0)  d B(a  1)  d  (a  3, 0, 0)  

( A, B, C ).(3  a, 0, 0)  d  A( a  3)  d . de donde A

d d d 1 d3 ,B  , C  además se tiene que:V  donde V  15u 3 a 3 a 1 a 6 ABC

d3 d d d  15  (a  3)(a  1)a  90  a  6 de donde A  , B  , C  d d d 3 5 6 . . a  3 a 1 a 111 como  : N .( x, y, z )  d   : d ( ).( x, y, z )  d 356

V

1 6

x y z   :   1 3 5 6 19. Hallar la ecuación de la recta que pasa por el punto (1,-1,1), perpendicular a la recta 3x = 2y = z y paralela al plano x + y – z = 0

Sean L : (1, 1,1)   ( a, b, c) /   R la recta buscada L1 : 3 x  2 y  z x y z 1 1    b  ( , ,1) 1 1 1 3 2 3 2 1 1 L  L1  (a, b, c)( , ,1)  0  2a  3b  6c  0 ...(1) 3 2 como el plano P: x  y  z  0, de donde N  (1,1, 1) por ser P // L  N  (a, b, c)  0 (1,1, 1).(a, b, c)  0 entonces a  b  c  0 ...(2) entonces: L1

2a  3b  0 a  9c ahora resolvemos el siguiente sistema:   a  b  c  0 b  8c 38

(a, b, c)  (9c, 8c, c)  c(9, 8,1) por lo tanto L  (1, 1,1)  (9, 8,1) /   R lo que es igual a expresar de la forma: L :

x 1 y  1 z 1   9 8 1

20. Sean 1 : 3x  y  z  1 y  2  x  y  3z  1, dos planos. Hallar las ecuaciones paramétricas de la recta L que pasa por las proyecciones del punto Q(1,1,1) sobre cada plano.

Del grafico se observa que la recta l pasa por los puntos Ay B que son las proyecciones del punto Q sobre cada plano, por lo tanto calcularemos los puntos A y B Para el punto A trazamos la recta L1, es decir: L1  (1,1,1)  t (3,1, 1) / t  R

como A  L1   1 entonces A  L1  A   1. Si A  L1  A(1  3t ,1  t ,1  t ) para algún t  R, ádemas A   1  3(1  3t )  1  t  t  1  1  t  

2 , 11

5 9 13 , , ). Para el punto B trazamos la recta L 2 , es 11 11 11 decir: L2  (1,1,1)  t (1, 1,3) / t  R como B  L 2   2  B  L 2  B   2 de donde el punto A(

Si B  L 2  B (1  t ,1  t ,1  3t ) para algún t  R además B   2  1  t  1  t  3(1  3t )  1  t  

2 11

9 13 5 , , ) 11 11 11 4 Sea a  AB  B  A  (1,1, 2) por lo tanto la recta L pedida es: 11

de donde el punto B(

 5 9 13  L  ( , , )   (1,1, 2) /   R  cuyas ecuaciones paramétricas son:  11 11 11 

5   x  11    9  L : y    11   13  z  11  2 

,  R

39

SUPERFICIES CUÁDRATICAS 1. INTRODUCCION Superficies la ecuación E(x,y) = 0 nos representa un lugar geométrico en el plano XY, la ecuación E(x,y) = 0, extenderemos al espacio tridimensional, cuya ecuación rectangular en tres variables representaremos por:

P  x, y, z   0 También se conoce que todo plano se representa analíticamente por una única ecuación lineal de la forma:

P : Ax  By  Cz  D  0 De una manera más general, veremos se existe una representación analítica de una figura geométrica, al cual denominaremos superficie, tal representación consistirá de una única ecuación rectangular de la forma:

F ( x, y, z)  0

(1)

Por Ejemplo, por medio de la distancia entre dos puntos se puede demostrar que la superficie esférica de radio r con centro en el origen se representa analíticamente por la ecuación:

x2  y 2  z 2  r 2 2. DEFINICIÓN 1. Definir una superficie, consiste en, caracterizarla por medio de una propiedad común a todos sus puntos. ( es decir expresarlo como un lugar geométrico) o por su ley de generación. Las superficies así definidas solo son estudiadas vía analítica, expresadas por la F x, y, z  0 a la cual satisfacen las coordenadas de ecuación: z  f  x, y o cada punto punto situado en esta superficie y no satisfacen las coordenadas de ningún otro punto situado fuera de ella a esta ecuación se denomina “ECUACION DE UNA SUPERFICIE”, la naturaleza de esta ecuación depende de la forma y posición de la superficie así como del sistema en el que se trabaja. En conclusión una superficie se representa por una sola ecuación de tres variables. A continuación presentamos algunas definiciones de Superficie: DEFINICIÓN: 2.- Supongamos un punto X cuyas coordenadas x , y , z sean funciones de dos parámetros u , v : x  x u, v , y  y u, v , z  z u, v (ecuaciones paramétricas de Superficie) Representando por la misma letra X al vector OX cuyas componentes son las coordenadas x , y , z , las tres ecuaciones paramétricas se pueden condensar en una ecuación vectorial única:    X  X u, v  x u, v i  y u, v j  z u, v k (ecuación vectorial de Superficie)

40

DEFINICIÓN: 3.- Sea A n un espacio afín de dimensión n; tomamos un origen O y la estructura vectorial V correspondiente. Se llama cuádrica al conjunto  de puntos x  A n tales que:     X   2   X   k  0 donde  es una forma cuadrática no nula de V ,  una forma lineal y k  K siempre que    La ecuación F(x,y,z) = 0 contiene tres variables, sin embargo la ecuación de una superficie pueden contener solamente una o dos variables. Por ejemplo la ecuación x = k constante, representa un plano paralelo al plano YZ.

De igual manera la ecuación x2 +y2 = 4 considerada en el espacio representa un cilindro circular recto.

Toda ecuación de la forma F(x,y,z) = 0, no necesariamente representa una superficie, por ejemplo la ecuación x2 +y2+z2+9 = 0, no representa ningún lugar geométrico, además la ecuación x2+y2+z2 = 0 tiene una sola solución real que es: x = y = z = 0, cuyo lugar geométrico esta constituido por un solo punto, el origen. 3. SUPERFICIES CUADRÁTICAS LUGARES GEOMÉTRICOS EN EL ESPACIO.El estudio de un lugar geométrico en el espacio corresponde, al igual que en el plano de dos fases distintas: i ) Mediante la definición del lugar geométrico el cual se deduce de sus ecuaciones o ecuación. ii ) Por el estudio de las propiedades algebraicas de éstas no solo la forma del lugar geométricos, sino también sus propiedades geométricas. El estudio de la geometría espacial de acuerdo al último análisis se reduce al estudio de una superficie; lo que podemos expresarlo de la siguiente manera: a ) Dada una superficie definida geométricamente; determinar su ecuación. b ) Dada una ecuación, determinar la superficie que representa. PRIMER PROBLEMA FUNDAMENTAL Este problema podemos resolverlo de la misma manera que en plano, por dos métodos: el directo y el indirecto. 41

LUGAR GEOMÉTRICO: Lamamos un lugar geométrico al conjunto de puntos del espacio que gozan de una misma propiedad expresada por la definición del lugar. Para demostrar que un lugar geométrico es una línea alabeada o superficie hay que probar: 1º Que todo punto del espacio que goza de la propiedad del lugar está sobre la línea o superficie. 2º Recíprocamente, todo punto situado sobre la línea o superficie considerada goza de la propiedad del lugar. La ecuación o ecuaciones existentes entre las coordenadas de los puntos de la línea o superficie en cuestión se llaman la ecuación o ecuaciones del lugar. Para obtener un lugar pueden aplicarse dos métodos generales: el directo y el indirecto. El método directo, cuando puede aplicarse, basta representar por variables x , y, z las coordenadas de un punto del lugar y traducir en seguida en forma algebraica, por medio de una o dos ecuaciones, la propiedad de que gozan todos los puntos del lugar. El método indirecto ( o de los parámetros ), que el más generalmente empleado, se consideran las superficies como lugares de líneas o generatrices obtenidas por intersección de dos superficies variables que dependen de uno o más parámetros y que se mueven con arreglo a leyes determinadas que condicionan estos parámetros cuando hay más de uno. Ejemplos: 1.- Determinar la ecuación del lugar geométrico de los planos cuya distancia al origen es igual al cuádruple de las abscisas respectivas. Solución Usando el método directo: Sea P  x, y, z un punto que satisface las condiciones dadas:  OP  4 x Pero:

OP  x 2  y 2  z 2

luego se tiene:

15 x 2  y 2  z 2  0 de donde: correspondiente.

4 x  x2  y2  z2

ecuación del lugar geométrico

2.- Determinar la ecuación de la superficie generada por la familia de curvas: x  2 y  k z  0 , y 2  k x , siendo k un parámetro variable. Solución Usando el método indirecto:  x  2 y x x2y de: x  2 y  k z  0  k  en y 2  k x  z z 2 2  y z  x  2 x y  0 es la ecuación de la superficie pedida. 3.- Determinar la ecuación de la superficie generada por la recta que pasa por el y2  z2  1 punto P 3,0,0 y se apoya en la circunferencia de ecuación:  x  0 Solución Usando el método indirecto:

42





Como la directriz es una línea recta  P1 x1 , y1 , z1  D la

generatriz

G

es:

 la ecuación de

x3 y z   x1  3 y1 z1

.

Además

P1  D  x12  z12  1 ; x1  0 ... (1) 3 y x3 y ............................................................(2)   luego y1  3 y1 x3 3 z x3 z .........................................................(3)   luego z1  3 z1 x3 2 2 Ahora (3) y (2) en (1) obtenemos: 9 y  9 z   x  3  0 2

SEGUNDO PROBLEMA FUNDAMENTAL: La determinación de la superficie representativa de una ecuación dada se procesa mediante el estudio sistemático de la misma, al cual se denomina: DISCUSIÓN DE LA ECUACIÓN. Para la discusión, caracterización y construcción de la superficie representativa de una ecuación, de manera análoga a como se hace en geometría plana se adoptará el siguiente procedimiento para construir la gráfica de una superficie consideremos la siguiente discusión, mediante los pasos siguientes: 1) 2) 3) 4) 5) 6)

Intersección con los ejes coordenados. Trazas sobre los planos coordenados. Simetrías con respecto a los planos coordenados, ejes coordinados y el origen. Secciones transversales o secciones paralelas a los planos coordenados. Extensión de la superficie. Construcción de la superficie. Consideremos la ecuación de una superficie.

F ( x, y, z )  0 Ahora describiremos todo el proceso a realizar en la construcción de la gráfica de dicha superficie. 1. Determinación de las intersecciones con los ejes coordenados: Para obtener la intersección de una superficie con determinado eje se anulan, en la ecuación de la superficie, las variables, las variables no correspondientes al eje considerado, obteniendo así, la coordenada del mismo nombre de su eje: x : en F  x , y , z   0 hacer z = y = 0

y : en F  x , y , z   0

z : en F  x , y , z   0

hacer x = z = 0 hacer x = y = 0

Si en F  x, y, z  0 se verifica x  y  z  0 , la superficie pasa por el origen. Esto sucede cuando no contiene términos independientes.

43

2. Determinación de las trazas sobre los planos coordenados: La traza de la superficie F  x, y, z  0 sobre el plano XY , por ejemplo en la intersección con el plano: Z = 0 . Haciendo, Z = 0 en F  x, y, z  0 se obtendrá f  x, y  0 ; que define en el plano XY, una curva que es la traza buscada. Podemos concluir que, para obtener la ecuación de la traza de una superficie con uno de los planos coordenados, bastará anular en la ecuación de la superficie la variable no correspondiente a los ejes del plano coordenado F  x, y, z  0 considerado. Esto es: i ) Traza sobre el plano XY :

 f  x , y   0 , hacer Z = 0 , y se obtiene:  z  0 ii ) Traza sobre el plano XZ :  f  x , z  0 en F  x, y, z  0 , hacer Y = 0 , y se obtiene:  y  0 iii ) i ) Traza sobre el plano YZ :  f  y , z  0 en F  x, y, z  0 , hacer X = 0 , y se obtiene:   x  0 Las trazas de una superficie sobre los planos coordenados se denominan: TRAZAS PRINCIPALES.

en F  x, y, z  0

3.- SIMETRÍA La simetría de una Superficie se considera en relación a los planos coordenados, a los ejes coordenados y al origen. i ) Simetría en relación a los planos coordenados: Si la ecuación de una superficie algebraica sólo tiene potencias pares de una de las variables, esa superficie es simétrica en relación al plano correspondiente a las otras dos variables. Esto es con respecto a los planos: XY debe cumplirse; F  x, y, z  F  x, y, z XZ debe cumplirse; F  x, y, z  F  x, y, z

YZ debe cumplirse; F  x, y, z  F   x, y, z ii ) Simetría en relación con los ejes coordenados: Si la ecuación de una superficie algebraica contiene potencias pares de dos variables e impares de la tercera variable, esa superficie es simétrica en relación a los ejes correspondiente a la tercera variables. Esto es con respecto a los ejes: X debe cumplirse : F  x, y, z  F  x, y, z Y debe cumplirse : F  x, y, z  F   x, y, z

Z debe cumplirse : F  x, y, z  F   x, y, z iii ) Simetría en el origen: Una superficie es simétrica en relación al origen cuando su ecuación algebraica solo contiene términos de grado par y de grado impar en relación a las variables. En la primera hipótesis la superficie es simétrica en relación a los ejes coordenados, a los planos coordenados y al origen, en la segunda, solo al origen. 44

4.- Sección y extensión: Las secciones planas se pueden obtener cortando la superficie por una serie de planos paralelos a los planos coordenados; por ejemplo los planos paralelos al plano XY, se obtienen de la ecuación Z = k ; k es una constante arbitraria.   F  x, y, k   0 Si F  x , y , z  0   .................................. ( * )  z  k ( * ) representa las ecuaciones de la curva de intersección del plano con las superficies, donde a cada valor de k le corresponde una curva. El campo de variación de k en (*) representa una curva real y define la extensión de la superficie en relación a los ejes coordenados, Sea aclara este asunto con el siguiente ejemplo: Ejemplo: 1.- Discutir la siguiente ecuación : x 2  y 2  z  4 ; Grafique. Solución 1º ) Intersecciones con los ejes: con F  x, y, z: x 2  y 2  z  4  0

x : en F  x , y , z   0

hacer z = y = 0  x =  z

y : en F  x , y , z   0

hacer x = z = 0  y =  z

z : en F  x , y , z   0 hacer x = y = 0  z = -4 2º ) Trazas: i ) Traza sobre el plano XY :

x 2  y 2  4 , hacer Z = 0 , y se obtiene:  Circunferencia z  0

en F  x, y, z  0

de: r = 2 ii ) Traza sobre el plano XZ :

x  z 2  4 en F x, y, z   0 , hacer Y = 0 , y se obtiene:  Parábola V(0,-4) = (x ,z) y  0 iii ) Traza sobre el plano YZ : y  z2  4 en F  x, y, z  0 , hacer X = 0 , y se obtiene:  Parábola x  0 de vértice V(0,-4) = (y ,z) 3º ) Simetría: La superficie es simétrica en relación a los planos ( XZ ) y ( YZ ) y en relación al eje Z 4º ) Sección y extensión: Para Z = k ; se tiene: x 2  y 2  4  k ( círculos de radio crecientes para k  0 y decrecientes para - 4  k  0 ). Si k = - 4 su radio es igual cero; las secciones paralelos al plano XY son imaginarios para k  - 4 ; la superficie no existe abajo del plano Z = - 4. 2 - Para y = k se tiene : x 2  z   4  k  son parábolas de vértices



v x, z  0, 4  k 

2



 k  R . Las parábolas son reales, luego la superficie se extiende indefinidamente a lo largo del eje X. 5º ) Esbozo de la imagen geométrica. GRAFICA : Con los elementos proporcionados por la discusión anterior, se puede hacer un esbozo de la superficie: 45

EJE Z

Seccion : Y Z

Seccion : X Z

EJE Y

Seccion : X Y

EJE X

Llamaremos superficies cuadráticas a toda ecuación de segundo grado en las variables x,y,z que tiene la forma: Ax2  By 2  Cz 2  Dxy  Exz  Fyz  Gx  Hy  Kz  L  0 donde A, B, C, D, E, F, G, H, K son constantes , y por lo menos una es diferente de cero. Ejemplo 2.- Discutir y hacer la gráfica de la superficie cuya ecuación es Solución A) Intersecciones con los ejes coordenados. a.

Con el eje X, se hace y = z= 0, de donde son: (1,0,0) , (-1,0,0)

b.

Con el eje Y, se hace x = z= 0, de donde son: (0,1,0) , (0,-1,0)

c. Con el eje Z, se hace x = y= 0, de donde B) Las trazas sobre los planos coordenados.

x2  y 2  z 2  1

x 2  1 entonces x  1 , de donde los puntos

y 2  1 entonces y  1 , de donde los puntos

z 2  1 entonces no existe intersección con el eje Z

x 2  y 2  1 es una circunferencia 2 2 b. La traza con el plano XZ: se hace y = 0; x  z  1 es una hipérbola a.

La traza con el plano XY: se hace z = 0;

c.

La traza con el plano YZ: se hace x = 0;

y 2  z 2  1 es una hipérbola C) Simetría con respecto a los planos coordenados, ejes coordenados y al origen. La superficie es simétrica al origen, a los ejes coordenados y a los planos coordenados, puesto que la ecuación no cambia al aplicar el criterio establecido. D) Las secciones transversales o paralelas a los planos coordenados: Consideramos las secciones paralelas al plano XY; sea z = k entonces de circunferencias. E) Extensión:

x 2  y 2  1  k 2 es una familia

z   x2  y 2  1 , x2  y 2  1

46

4. DISCUCIÓN DE LAS PRINCIPALES SUPERFICIES CUADRÁTICAS Una superficie muy común es la dada por una ecuación de la forma A x2  B y2  C z 2  D x  E y  F z  G  0 que denominamos superficie cuádratica o simplemente cuádrica. Veremos la discusión de las cuádricas de aquellas cuyo centro está en el origen de coordenadas y cuyos ejes siguen la dirección de los ejes coordenados. Las seis cuádricas fundamentales son: i) ii) iii) iv) v) vi)

Elipsoide Hiperboloide de una hoja Hiperboloide de dos hojas Paraboloide Elíptico Paraboloide Hiperbólico Cono

La manera más sencilla de representar el gráfico de una cuádrica es hallar sus intersecciones con los ejes y determinar las secciones producidas por cada uno de los planos coordenados o planos paralelos a los coordenados Es el lugar de todos los puntos p(x,y,z) de R 3 que x2 y 2 z 2 satisfacen a la ecuación de la forma: 2  2  2  1 , a b c a  0, b  0 , c  0, a  b, a  c ó b  c. Graficando el elipsoide se tiene: a) Intersecciones con los ejes coordenados - Con el eje X, se hace y  z  0, x  a , A1  a,0,0  , A2  a,0,0  1) ELIPSOIDE.-

-

Con el eje Y, se hace

x  z  0, y  b , B1  0, b,0  , B2  0, b,0 

x  y  0, z  c , C1  0,0, c  , C2  0,0, c  b) Las Trazas sobre los planos coordenados. x2 y 2 - La traza sobre el plano XY, se hace z = 0 2  2  1 , es una elipse en el plano XY a b 2 x z2 - La traza sobre el plano XZ, se hace y = 0 2  2  1 , es una elipse en el plano XZ a c y2 z2 - La traza sobre el plano YZ, se hace x = 0 2  2  1 , es una elipse en el plano YZ b c c) Simetrías con respecto al origen, ejes y planos coordenados. x2 y 2 z 2 Sea E : 2  2  2  1 , entonces a b c - Con respecto al origen ; sí  a, b, c      x,  y,  z  

-

Con el eje X, se hace

-

Con respecto al eje X

; sí  a, b, c     x,  y,  z  

-

Con respecto al eje Y

; sí  a, b, c      x, y,  z  

-

Con respecto al eje Z

; sí  a, b, c      x,  y, z  

-

Con respecto al plano XY

; sí  a, b, c     x, y,  z   47

-

Con respecto al plano XZ

; sí  a, b, c     x,  y, z  

; sí  a, b, c      x, y, z   d) Las secciones paralelas a los planos coordenados. x2 y 2 k2 Los planos z = k, corta la superficie en la curva 2  2  1  2 , que es una familia a b c de elipses donde c  k  c . -

Con respecto al plano YZ

e) Extensión de la superficie de

x2 y 2 x2 y 2 z 2 z  | c | 1   se tiene    1 a 2 b2 a 2 b2 c 2

x2 y 2 de donde 2  2  1 a b

2) Esfera.- La superficie es el lugar geométrico de todos los puntos p(x,y,z) en el espacio que equidistan de un punto fijo, la distancia constante se llama radio y el punto fijo centro.

x2 y 2 z 2 Si la ecuación del elipsoide 2  2  2  1 se tiene a = b = c  0, el a b c 2 2 2 elipsoide se transforma en x  y  z  R2 , que es la ecuación de la esfera de radio R y centro en el origen de las coordenadas. Graficando la esfera se tiene: a) Intersecciones con los ejes coordenados. - Con el eje X, se hace, y  z  0 , x   R , A1  R,0,0  , A2   R,0,0  -

Con el eje Y, se hace,

x  z  0 , y   R , B1  0, R,0  , B2  0,  R,0 

x  y  0 , z   R , C1  0,0, R  , C2  0,0,  R  b) Las trazas sobre los planos coordenados. -

Con el eje Z, se hace,

-

La traza sobre el plano XY, se hace z = 0.

x 2  y 2  R 2 , es una circunferencia del plano XY -

La traza sobre el plano XZ, se hace y = 0

x2  z 2  R2 , es una circunferencia en el plano XZ -

La traza sobre el plano YZ, se hace x = 0

y 2  z 2  R2 , es una circunferencia en el plano YZ c)

Simétricas respecto al origen, ejes y planos coordenados. 2 2 2 2 La ecuación de la esfera x  y  z  R es simétrica respecto al origen, a los ejes y planos coordenados.

d) Las secciones paralelas a los planos coordenados. 48

Las secciones paralelas lo tomaremos con respecto al plano coordenado XY, es decir, Z = K se tiene x circunferencia.

2

 y 2  R2  k 2 ,  R  k  R , que es una familia de

TEOREMA.- La ecuación de la superficie esférica con centro en el punto 2 2 2 c(h,k,l) y de radio la constante R > 0 es:  x  h    y  k    z  l   R 2 Demostración Sea P(x,y,z) un punto cualquiera de la esfera, luego por definición de esfera se tiene: E  P  x, y, z   R3 / d ( p, c)  R

 x  h   ( y  k )2  ( z  l )2  R 2 2 2  x  h    y  k    z  l   R2 2

de donde:

OBSERVACIÓN.- La ecuación  x  h    y  k    z  l   R 2 se conoce con el nombre de forma ordinaria de la ecuación de la esfera, si desarrollamos la ecuación de la esfera se tiene: x2  y 2  z 2  2hx  2ky  2lz  h2  k 2  l 2  R 2  0 , de donde se tiene: 2

3)

2

2

x2  y 2  z 2  Ax  By  Cz  D  0

PARABOLOIDE ELÍPTICO.- Es el lugar geométrico de todos los puntos p(x,y,z) de R3 que satisfacen la x2 y 2 ecuación de la forma 2  2  z , de donde a  0, b  0, a  b a b Graficando el paraboloide elíptico tenemos: a) Intersecciones con los ejes coordenados. - Con el eje X, se hace, y  z  0 , x  0 , A(0,0,0) - Con el eje Y, se hace, x  z  0 , y  0 , B(0,0,0) - Con el eje Z, se hace, x  y  0 , z  0 , C (0,0,0) b) Las trazas sobre los planos coordenados -

-

La traza sobre el plano XY, se hace z = 0

x2 y 2   0 que representa un punto a 2 b2 La traza sobre el plano XZ, se hace y = 0

z -

x2 que representa a una parábola en el plano XZ a2

La traza sobre el plano YZ, se hace x = 0

y2 z  2 que representa a una parábola en el plano YZ b c) Simetrías respecto al origen, ejes y planos coordenados. - Con respecto al origen  puesto que   x,  y,  z   P 49

-

Con respecto al eje X,

 puesto que  x,  y,  z   P

-

Con respecto al eje Y,

 puesto que   x, y,  z   P

-

Con respecto al eje Z,

 puesto que   x,  y, z   P

-

Con respecto al plano XY,

 puesto que  x, y,  z   P

-

Con respecto al plano XZ,

 puesto que  x,  y, z   P

 puesto que   x, y, z   P d) Secciones paralelas a los planos coordenados. -

Con respecto al plano YZ,

Las secciones paralelas tomaremos con respecto al plano XY para esto se tiene z = k

x2 y 2 que corta en la superficie en la curva 2  2  k que es de la familia de elipses a b 2 2 x y e) Extensiones de la superficie: z  2  2 es definido ( x, y)  R2 a b

OTRAS VARIANTES

4)

HIPERBOLOIDE DE UNA HOJA.- Es el lugar geométrico de todos los puntos P(x,y,z) de R3 que x2 y 2 z 2 satisfacen a la ecuación 2  2  2  1 , donde a  0 , b  0 , c  0 . a b c Graficando el hiperboloide de una hoja se tiene. a) Intersecciones con los ejes coordenados - Con el eje X, se hace y  z  0 , x  a , A1  a, 0, 0  A2  a,0,0  -

Con el eje Y, se hace

x  z  0 , y  b , B1  0, b,0  , B2  0, b,0 

-

Con el eje Z, se hace

x  y  0 , z 2  c 2 , 

50

b) Las trazas sobre los ejes coordenados.

x2 y 2   1 , es elipse a 2 b2 x2 z 2 - La traza sobre el plano XZ, se hace y = 0; donde 2  2  1 , es hipérbola a c y2 z2 - La traza sobre el plano YZ, se hace x = 0; donde 2  2  1 , es hipérbola b c c) Simetrías

d)

-

La traza sobre el plano XY, se hace z = 0; donde

-

Con respecto al origen es simétrica. Con respecto a los ejes coordenados es simétrica Con respecto a los planos coordenados es simétrica

Secciones paralelas a los planos coordenados Los planos z = k corta a la superficie en la curva

x2 y 2 k2   1  2 , que es una familia a 2 b2 c de elipses y los planos y = k corta a la superficie en la curva

x2 z 2 k 2 b2  k 2 ,   1   a2 c2 c2 b2 b  k  b , que es una familia de hipérbola. OTRAS VARIANTES

5) HIPERBOLOIDE DE DOS HOJAS.-

Es el lugar geométrico de todos los puntos P 2 2 2 x y z (x,y,z) de R3 que satisfacen la ecuación: 2  2  2  1 , donde a ≠ 0, b ≠ a b c 0, c ≠ 0. Graficando el hiperboloide de dos hojas se tiene: a) Intersecciones con los ejes coordenados - Con el eje X, se hace y  z  0 , x  a , A1  a, 0, 0  A2  a,0,0  -

Con el eje Y, se hace

x  z  0 , y   b2 , 

-

Con el eje Z, se hace

x  y  0 , z   c 2 , 

51

b) Las trazas sobre los ejes coordenados.

x2 y 2   1 , es hipérbola a 2 b2 x2 z 2 - La traza sobre el plano XZ, se hace y = 0; donde 2  2  1 , es hipérbola a c y2 z2 - La traza sobre el plano YZ, se hace x = 0; donde  2  2  1,  b c c) Simetrías -

La traza sobre el plano XY, se hace z = 0; donde

-

Con respecto al origen existe simetría. Con respecto a los ejes coordenados, existe simetría Con respecto a los planos coordenados es simétrica

d) Secciones paralelas a los planos coordenados

x2 y 2 k2 Los planos z = k, corta a la superficie, dando la curva 2  2  1  2 que es una a b c familia de hipérbolas. Los planos y = k, corta a la superficie, dando la curva

x2 z 2 k2   1  que es una a2 c2 c2

familia de hipérbolas. Los planos x = k, corta la superficie dando la curva

y2 z 2 k 2  a2   donde K > a b2 c 2 a2

ó k < -a, que es una familia de elipses.

52

OTRAS VARIANTES

6) HIPERBOLOIDE PARABOLICO.- Es el lugar geométrico de todos los y 2 x2 z puntos P (x,y,z) de R3 que satisfacen la ecuación: 2  2  , donde a y b b a c son positivos y c ≠ 0. Graficando el hiperboloide parabólico para el caso c > 0. a) Intersecciones con los ejes coordenados - Con el eje X, se hace y  z  0 , x  0 A  0,0,0  -

x  z  0 , y  0, B(0,0,0) Con el eje Z, se hace x  y  0 , z  0, C(0,0,0) Con el eje Y, se hace

b) Las trazas sobre los ejes coordenados.

b b x, y   x , rectas. a a 2 c 2 - La traza sobre el plano XZ, se hace y = 0; z   2 x , parábola. a c2 2 - La traza sobre el plano YZ, se hace x = 0; z   2 y , parábola. b c) Simetrías - Con respecto al origen  y

-

La traza sobre el plano XY, se hace z = 0;

-

Con respecto a los ejes coordenados, con el eje Z

 en los demás eje 

 Pxy , Pxz , Pyz d) Secciones paralelas a los planos coordenados y2 x2 k - Al plano XY, se hace z = k; 2  2  , familia de hipérbolas. b a c x2 z k 2 - Al plano XZ, se hace y = k;  2   2 , familia de parábola. a c b -

Con respecto a los planos coordenados

53

-

Al plano YZ, se hace x = k;

y2 z k 2 , familia de parábola.   b2 c a 2

OTRAS VARIANTES

7) EL CONO ELÍPTICO O CIRCULAR.- Es el lugar geométrico de todos los puntos P (x,y,z) de R3 que satisfacen x2 y 2 z 2 la ecuación: 2  2  2 , a ≠ 0, b ≠ 0, c ≠ 0. a b c Graficando el cono elíptico. a) Intersecciones con los ejes coordenados - Con el eje X, se hace y  z  0 , x  0 A  0,0,0  - Con el eje Y, se hace x  z  0 , y  0, B(0,0,0) - Con el eje Z, se hace x  y  0 , z  0, C(0,0,0) b) Las trazas sobre los ejes coordenados. - La traza sobre el plano XY, se hace z = 0; x = y = 0  p(0,0,0). a - La traza sobre el plano XZ, se hace y = 0; x   z dos rectas. c b - La traza sobre el plano YZ, se hace x = 0; y   z dos rectas. c 54

c) Simetrías -

Con respecto al origen existe. Con respecto a los ejes coordenados, existe Con respecto a los planos coordenados existe

d) Secciones paralelas a los planos coordenados x2 y 2 k 2 - Al plano XY, se hace z = k; 2  2  2 , familia de elipses. a b c 2 2 z x k2 - Al plano XZ, se hace y = k; 2  2  2 , familia de hipérbolas. c a b 2 2 2 z y k Al plano YZ, se hace x = k; 2  2  2 , familia de hipérbolas. c b a

OTRAS VARIANTES

5. SUPERFICIES CILÍNDRICAS Llamaremos superficies cilíndrica a la superficie que es generada por una recta que se mueve a lo largo de una curva plana dad, de tal manera que siempre se mantenga paralela a una recta fija dad que no está en el plano de dicha curva. La recta móvil se llama generatriz y la curva plana se llama directriz de la superficie cilíndrica. Si la generatriz de una superficie cilíndrica es perpendicular al plano de la directriz; la superficie se llama cilindro recto, en caso contrario cilindro oblicuo.

55

6. DETERMINACIÓN DE LA ECUACIÓNDE UNA SUPERFICIE CILÍNDRICA Consideramos la directriz en uno de los planos coordenados por ejemplo, tomamos el plano YZ, entonces la ecuación de la directriz F(y,z)  0 es: D :  x0  Si p(x, y, z) es un punto cualquiera de la superficie, cuya generatriz tiene por números directores [a, b, c] y si p’(0, y’, z’) es el punto de intersección de la directriz que pasa por el punto p(x,y,z) entonces el punto p’(0,y’,z’) satisface a la ecuación de la directriz F(y', z')  0 D: x'  0 

… (1)

Y la ecuación de la Generatriz es dado por:

G:

x0 y  y' z  z'   a b c

… (2)

De las ecuaciones (1) y (2) al eliminar x’, y’, z’ se tiene la ecuación de la superficie cilíndrica. 7. SUPERFICIE CÓNICA Llamaremos superficie cónica a la superficie que es generada por una recta que se mueve de tal manera que siempre pasa por una curva plana dada fija y por un punto fijo que no está contenido en el plano de la curva fija dada. La recta móvil se llama generatriz y la curva fija dada directriz y el punto fijo se llama vértice de la superficie cónica. El vértice divide a la superficie cónica en dos porciones cada una de los cuales se llama hoja o rama de la superficie cónica

56

8. DETERMINACIÓN DE LA ECUACIÓN DE LA SUPERFICIE CÓNICA Consideremos la ecuación de la directriz en uno de los planos coordenados, por ejemplo en el F(y,z)  0 plano YZ, cuya ecuación es D :  y el x0  vértice V ( x0 , y0 , z0 ) . Como P’(x’,y’,z’) pertenece a la directriz, por lo tanto lo satisface, es decir: F(y', z')  0 D: x'  0 

... (1)

La ecuación de la generatriz que pasa por V y p’ es dado por: G:

x  x0 y  y0 z  z0   x ' x0 y ' y0 z ' z0

…(2)

De las ecuaciones (1) y (2) al eliminar los parámetros x’, y’, z’ se obtiene la ecuación de la superficie cónica. 9. SUPERFICIES DE REVOLUCIÓN Llamaremos Superficie de revolución a la superficie que es generada por la rotación de una curva plana de una recta fija contenida en el plano de esa curva. La curva plana se llama generatriz y la recta fija eje de revolución ó eje de la superficie.

Por el punto p(x, y, z) se hace pasar un plano perpendicular al eje de revolución, la intersección de la superficie con el plano es una circunferencia. Si c es el punto de intersección del plano con la recta L y Q es el punto de intersección con la curva C entonces se cumple d(P,C) = d(Q,C) que es la ecuación de la superficie de revolución. Si la superficie de revolución es obtenida por la rotación de una curva que está en uno de los planos coordenados alrededor de uno de los ejes coordenados, su ecuación se determina mediante el siguiente cuadro:

57

Ecuación de la Generatriz z = f(y); x = 0

Eje de Revolución Eje Y

Ecuación de la superficie

x2  z 2   f ( y) 

2

x = f(y); z = 0

Eje Y

x2  z 2   f ( y) 

2

z = f(x); y = 0

Eje X

y 2  z 2   f ( y) 

2

y = f(x); z = 0

Eje X

y 2  z 2   f ( y) 

2

y = f(z); x = 0

Eje Z

x2  y 2   f ( y) 

2

x = f(z); y = 0

Eje Z

x2  y 2   f ( y) 

2

Ejemplo: Hallar la ecuación de la superficie de revolución engendrada al girar la curva dada alrededor del eje señalado. E : x y  2 en el plano y z al rededor del eje y. Solución Sabemos que: x y  2 es una curva en el plano x y cuyas ecuaciones son: ............................................(1) f x, y  0 , z  0 también que si una curva rota alrededor del eje y entonces la función

 

  

x  z  f y ....................(2) generatriz tiene la forma: Además P x, y, z  S y que el paralelo que pasa por P corta a la generatriz G en un punto del plano: x y es P x, y , z  y su centro es C que pertenece al eje x ( ver figura ) . Por ser radios del mismo paralelo se tiene que: 2







CP  CP  pero CP  y 

CP 

2

2



y 2  z 2 por la ecuación (2) y también

 y   y 2  z 2

................................................(3)

Además P  P están en el mismo plano entonces: x  x .......................................................................(4) Como P  G (generatriz que tiene la forma de la ecuación (1)  f x, y   0 ; z   0 ................................. (5) Por eliminación de parámetros x , y  , z  entre la ecuaciones 1,2,3,4,5





Obtenemos: f  x ,  y 2  z 2   0   Luego reemplazamos en la curva dada: x y  2 se tiene:

x y2  z2  2  y2  z2 





 x2 y2  z2  4

4 x2

El cual representa un cono 10. TRASLACIÓN DE EJES La Traslación de ejes en el espacio tridimensional se realiza en forma similar que la traslación en el plano cartesiano; si O’(x0, y0, z0) es un punto en el sistema cartesiano OXYZ, entonces en el punto O’(x0, y0, z0) construiremos el nuevo sistema o’x’y’z’ de tal manera que los rayos positivos de los nuevos ejes sean 58

paralelos y tengan el mismo sentido que el sistema cartesiano original, es decir, en la forma:

Un punto p en el espacio correspondiente al sistema OXYZ, se tiene por coordenadas a (x, y, z) es decir p(x, y, z) y el sistema O’X’Y’Z’ tiene por coordenadas a (x’, y’, z’) es decir p(x’, y’, z’). La relación entre estas coordenadas esta dada por:

 x  x0  x   y  y0  y  z  z  z 0  11. ROTACIÓN DE LOS EJES EN UNO DE LOS PLANOS COORDENADOS Veremos la rotación de los ejes de los planos coordenados manteniéndose el otro fijo y el mismo origen. Suponiendo que efectuamos una transformación de coordenadas del plano XY en otro sistema X’Y’ en donde se mantiene fijo el origen y los ejes X’ e Y’ son obtenidos rotando los ejes X e Y en forma antihoraria en un ángulos  como se ilustra en la figura.

Esta transformación en el plano XY es:

Cada punto p tendrá dos representaciones una en coordenadas (x,y) con respecto al sistema original otras en coordenadas (x’, y’) con respecto al nuevo sistema. 59

Ahora determinaremos la relación (x,y) y(x’, y’), para esto tracemos las rectas OP, AP y BP (ver figura)

Se observa que x  OA , y  AP , x  OB , y  BP Luego el triangulo OAP se tiene: x  OP cos     , y  OPsen     , donde

  x  OP cos  cos   OPsen sen    y  OPsen cos   OPsen cos 

…(1)

En el triangulo OBP se tiene: x  OP cos  , y  OPsen …(2)  x  x cos   ysen Ahora reemplazando (2) en (1) se tiene  al resolver el sistema  y  xsen  y cos   x  x cos   ysen se tiene: …(3)   y  x cos   ysen Por tratarse del plano XOY veremos el caso de la ecuación de segundo grado: Ax2  Bxy  Cy 2  Dx  Ey  F  0 , donde A, B, C no nulos simultáneamente. Como x  x cos  ysen , y  xsen  y cos se tiene: A( x cos   ysen ) 2  B( x cos   ysen )( xsen  y cos  )  C ( xsen  y cos  ) 2  D( x cos   ysen )  E ( xsen  y cos  )  F  0

B cos 2   A  C  sen2  ctg 2 

cos 2 A  C  sen2 B

AC B

desarrollado y simplificando se tiene:

 A cos   Bsen cos  Csen   x   Asen   Bsen cos  C cos   y 2

2

2

2

2

2



 B cos 2  Asen2  Csen2  xy  D  cos   Esen  x   E cos  D cos  y  F  0 Como el coeficiente de x’y’ debe ser cero, entonces se tiene: B cos 2  Asen2  Csen2  0 de donde: cos 2 A  C AC B cos 2   A  C  sen2   por lo tanto ctg 2  que es la sen2 B B relación para obtener el ángulo de rotación.

60

Rotación de los ejes coordenados rectangulares en el espacio: Si hacemos girar los ejes coordenados rectangulares en torno de su origen “O” como punto fijo de manera que los ángulos directores de los nuevos ejes x , y  , z  con respecto a los originales x , y , z sean:  1 ,  1 ,  1 ;  2 ,  2 ,  2 ;  3 ,  3 ,  3 , respectivamente, y las coordenadas de un punto cualquiera P del espacio antes y después de la rotación son x, y, z y x, y , z  respectivamente, entonces las ecuaciones de transformación de las originales a las nuevas son:









 x  x  cos  1  y  cos  2  z  cos  3  T :  y  x  cos  1  y  cos  2  z  cos  3 ....................................................(1)  z  x  cos   y  cos   z  cos  1 2 3  y las ecuaciones de la transformación inversa de las coordenadas nuevas a las originales son:

 x  x cos  1 y cos  1 z cos  1  T  :  y   x cos  2  y cos  2  z cos  2 ....................................................(2)  z   x cos   y cos   z cos  3 3 3  Se observa que el sistema formado por (1) incluye 9 coeficientes variables, que son los cosenos de los ángulos directores de los nuevos ejes en relación a los antiguos. Estos coeficientes no son entretanto independientes, estando ligados por las siguientes relaciones: cos 2  1  cos 2  1  cos 2  1  1   T cos 2  2  cos 2  2  cos 2  2  1 .........................................................................(3)  2 cos  3  cos 2  3  cos 2  3  1  

y , por ser los ejes perpendiculares entre si:

cos  1 cos  2  cos  1 cos  2  cos  1 cos  2  0  T  : cos  1 cos  3  cos  1 cos  3  cos  1 cos  3  0 cos  cos   cos  cos   cos  cos   0 2 3 2 3 2 3 

, , ,

x  y  x   z  ........(4)   y   z  

De manera que es necesario que se den por lo menos tres ángulos directores para que el sistema esté constituido por las fórmulas de transformación queda determinado y se pueden calcular las nuevas coordenadas en función de las antiguas ó viceversa. Transformación General Combinando los dos casos anteriores, este es, una translación de los ejes que transporte el origen hacia el punto O x0 , y0 , z 0 y una rotación de manera que los ángulos directores de los ejes finales O x , O y  , O z  en relación a los semiejes O x , O y  , O z  sean los indicados en el artículo anterior se tiene:





 x  x0  x cos  1 y cos  2  z cos  3  T :  y  y0  x cos  1 y cos  2  z cos  3 ...................................................(5)  z  z  x cos   y cos   z cos  0 1 2 3 

61

Obs: Como las fórmulas generales de transformación de coordenadas son funciones lineales de las variables, se concluye que la ecuación de una superficie o de una línea en el espacio no cambiará de grado o de orden ante cualquier transformación que sufran los ejes coordenados. COORDENADAS CILÍNDRICAS Y ESFÉRICAS El sistema de coordenadas cilíndricas, un punto en el espacio tridimensional está representado por la Terna coordenada donde y son las coordenadas polares de la proyección de en el planos al punto .

Para convertir de coordenadas cilíndricas a rectangulares, emplearemos las ecuaciones:

x  rCos y  rSen zz Mientras que para convertir de coordenadas rectangulares a cilíndricas usamos:

r 2  x2  y2 y Tan  x zz Ejemplo: 1.- Determine el punto con coordenadas rectangulares.

y encuentre sus coordenadas

x  2Cos120  1 y  2 Sen120  1.732  3 z 1 P  (1, 3 ,1)

2.- Encuentre las coordenadas cilíndricas del punto (3,-3,-7)

 3   3 

  Tan 1    45 r

3

2

  3

2



r  18  4.2426 z  7 P

 18 , / 4,7

62

COORDENADAS CILÍNDRICAS Las coordenadas cilíndricas son útiles en problemas que tienen simetría alrededor de su eje, en ese caso se selecciona el eje z de manera que coincida con el eje de simetría. 2 2 2 Por ejemplo el eje de un cilindro circular con ecuación cartesiana x  y  c es el eje . En las coordenadas cilíndricas, este cilindro tiene ecuación r  c . Esta es la razón del nombre que recibe como coordenadas cilíndricas.

Ejemplo: 2 2 2 3.- Encuentre la ecuación en coordenadas cilíndricas de un elipsoide 4 x  4 y  z  1 . 2 2 2 Sabemos que x  y  r

Retomando la ecuación de la elipsoide

z 2  1 4x2  4 y2



z 2  1 4 x2  y2



z 2  1  4r 2  como  r  c z 2  1  4c 2 Ecuación del elipsoide en coordenadas cilíndricas. COORDENADAS ESFÉRICAS Las coordenadas esféricas

de un punto

en el espacio

Un sistema de coordenadas esféricas se aplica en un problema donde hay simetría alrededor de un punto y el origen se pone en ese punto. Por ejemplo, la esfera con centro en el origen de radio tiene la ecuación ; esta es la razón por la cual reciben el nombre de coordenadas esféricas. Sabemos que x  rCos y y  rSen ; de modo que para convertir de coordenadas esféricas a rectangulares empleamos las ecuaciones:

x  SenCos y  SenSen z  Cos 2 2 2 2 Del mismo modo, la fórmula de la distancia muestra que   x  y  z .

63

Ejemplo: 4.- Dado el punto rectangulares.

2,  4 ,  3 . Hallar el punto y encontrar sus coordenadas

x  SenCos x  2Sen60Cos 45 x  1.2247

y  SenSen z  Cos y  2Sen 60Sen 45 z  2Cos 60 y  1.2247 z 1

Re c 3 , 3 ,1 2   2





5.- El punto 0,2 3,2 está dado en coordenadas rectangulares. Encuentre sus coordenadas esféricas.

  0 2  2 3    22 2

  12  4   16  4

z  Cos 2   Cos 1 4   60

PROBLEMAS DE SUPERFICIES EN R3 1. Hallar la ecuación de la esfera que están en los planos paralelos 6 x  3 y  2 z  35  0 , 6 x  3 y  2 z  63  0 . Sabiendo que el punto P(5,-1,-1) es el punto de contacto de uno de ellos. Solución

Sea L   5, 1, 1  t (6, 3, 2) / t  R Sea A  L  P2  A  L  A  P2 Si A  L  A  5  6t , 1  3t , 1  2t 

A  P2 entonces: 6  5  6t   3  1  3t   2  1  2t   63  0 de donde: t  2 , A(7,5,3) , como c es punto medio de Ay p  5  7 1  5 1  3  se tiene: c  , ,   c  1, 2,1 2 2   2 Para algún

Además, r = d(c,p) =7 por lo tanto:

t  R , como

E :  x  1   y  2    z  1  49 2

2

2

64

2. Hallar la ecuación del plano tangente a la esfera x2 +y2 + z2 = 49 en el punto M(6,-3,-2) SOLUCION

OM // N pero OM  M  O   6, 3, 2  Luego N   6, 3, 2  entonces la ecución del plano tangente en M será; P : N .  x  6, y  3, z  2   0  P : 6 x  3 x  2 z  49  0 3. Demostrar que el plano 2x – 6y + 3z – 49 = 0, es tangente a la esfera x2 + y2 + z2 = 49. Calcular las coordenadas del punto de contacto SOLUCION Si P : 2 x  6 y  3z  49  0 es tangete a la esfera

x 2  y 2  z 2  49 entonces d  C , P   r C : Centro de la esfera   0, 0, 0  r : radio de la esfera  7 | 2(0)  6(0)  3(0)  49 | 49 d C, P    7 4  36  9 49 Por lo tanto P es tangente al Plano Para hallar el punto de contacto. Hallamos la recta que pasa por el centro y el punto de contacto, que por definición tendrá como vector direccional el vector normal del plano tangente: L  t  2, 6,3 / t  R intersectando L con el plano 2(2t )  6(6)  3(3t )  49  0  4t  36t  9t  49  t  1

 P0   2, 6,3 4. Hallar la ecuación del cilindro cuyas generatrices son paralelas al vector  x2  y 2  9 a   2, 3, 4  , si las ecuaciones de la directriz son:  z  1 SOLUCIÓN  x2  y2  9 Sea D :  . la directriz. z  1 Sea P´ x´, y´, z´  D

 x '2  y '2  9 entonces la satiface D :  z '  1 ahora calculamos la generatriz, es decir: x  x' y  y' z  z' G:   , 2 3 4 x  x ' y  y ' z 1 de donde G :   ...(2) 2 3 4

...(1)

65

Eliminando los parámetros x’, y’ de la ecuaciones (1) y (2) 2x  z  1  x  x ' z 1   2  4  x '  2 Luego de la ecuacion (2) se tiene    ...(3)  y  y '  z 1  y '  4 y  3z  3 4  4  3 reemplazamos (3) en (1) y se tiene:  2 x  z  1   4 y  3z  3       49 2 4     2

2



4  2 x  z  1   4 y  3z  3  144 2

2

 16 x2  16 y 2  13z 2  16 xz  24 yz  16 x  24 y  26 z  131 5. Hallar la ecuación de la esfera que pasa por las circunferencias x2  z 2  25, y  2; x2  z 2  16, y  3 SOLUCIÓN Con los datos haremos un gráfico para visualizar el planteamiento del problema Del grafico se tiene A(0,2,5) en el Triangulo ACD. 2 2 r 2   2  b   252 también r 2   3  b   16 por tanto al igualar se tiene.

 2  b

2

 252   3  b   16  b  2 2

Luego el centro es C  0.  2, 0  y el radio r 2  41  x 2   y  2   z 2  41 6. Hallar la ecuación de la esfera que pasa por las circunferencias x2 +y2 =25, z = 2; x2 +y2 =16, z = 3 SOLUCIÓN 2

Sea C el centro de la esfera de radio R luego en el ACD : R 2   3  a   25 2

En el CBE : se obtiene: R 2   3  a   16 Luego igualando se tiene: 2 2  2  a   25   3  a   16 2

 2a  4 de donde a  2 Luego el centro es C(-2,0,0) y el radio R2 = 41 por lo tanto la ecuación de la esfera es: 2  x  2  y 2  z 2  41 7. Encontrar la ecuación del cono, con vértice en el origen, cuyas generatrices hacen un ángulo de 300 con el vector unitario que forman ángulos iguales con los ejes X, Y, Z. SOLUCIÓN

66

Por datos del problema se tiene: u= (cos ,cos ,cos ) como cos 2  cos 2   cos 2  1  3cos 2  1 puesto que cos  cos  cos 3 3  u 1,1,1 2 3

cos   

Sea r   x, y, z  el vector de posición de un punto cualquiera del cono, como por dato se tiene entonces. r .u || r || . || u || cos 30o

 x, y , z 

3 3 1,1,1  x 2  y 2  z 2 . 3 2

1  x  y  z  3

x2  y 2  z 2 2

 2  x  y  z   3 x2  y 2  z 2

8. Hallar la ecuación del cono que tiene el vértice en el punto (0,0,C); si las ecuaciones x2 y 2 de la directriz son 2  2  1 , z = 0 a b  x2 y 2 1   Sea D :  a 2 b 2 , la curva directriz. Si P '  x, y, z   D z  0   x '2 y '2  1  entonces lo satisface D :  a 2 b 2 ...(1) z '  0 

Ahora calcularemos la ecuaci{on de la generatriz donde V  0,0,C  es el vértice de la

x0 y 0 z c  y como z'  0 x ' 0 y ' 0 z ' c 9. Una vez comprobado que el punto M(1,3,-1) esta situado en el paraboloide hiperbólico superficie cónica. G :

4x2 – z2 = y, hallar las ecuaciones de sus generatrices que pasa por el punto M. SOLUCIÓN Sea H : 4 x 2  z 2  y  M 1, 3, 1  H

 4 1  3

y ...(1) k y L2 : 2 x  z  k  2 x  z  ...(2) k de la ecuación (1) 2 x  z  k  2 1  k  k  1 x y 1 z 1 L1 : 2 x  z  1  2 x  z  y  L1 :   1 4 2 de la ecuación(2), 2 x  z  k  2  1  k  k  3 y L2 : 2 x  z  3  2 x  z   L2 : z  2 x  3  y  12 x  9 3  x, y, z   L2   x, y, z    x,12 x  9, 2 x  3   0, 9, 3   x 1,12, 2  de donde:

L2 :

L1: 2 x  z  k  2 x  z 

x y9 z 3   1 12 2

67

10. Hallar la ecuación del cono que tiene el vértice en el origen de coordenadas, si las

 x2  2 z  1  0 ecuaciones de la directriz son:   y  z 1  0 SOLUCION

 x2  2 z  1  0  x '2  2 z ' 1  0 Sea D :  sí P'(x',y',z')  D :  ...(1) y  z  1  0 y '  z '  1  0   La ecuación de la generatriz es: G :

x0 y0 z 0   x ' 0 y ' 0 z ' 0

x y z   .....(2) x' y' z' de las ecuaciones (1) y (2) eliminamos los parámetros x',y',z' de donde G :

x z xz '   x'  z '  x '  z y z     y '  yz '   y z ' z remplazando (3) en la ecuación y'+z'+1=0

Ahora reemplazamos (4) en x'-2z'+1= 0, se tiene: (

x 2 2z )  1  0 z-y zy

simplificando tenemos la ecuación x 2  z 2  y 2  0 11. Las generatrices de un cilindro circunscrito en la esfera x 2 + y2 +z2 = 1 son perpendiculares al plano x + y – 2z + 5 = 0. Hallar la ecuación de este cilindro. SOLUCION Considerando la curva directriz en el plano XY para esto z = 0, por lo tanto, la directriz

 x2  y 2  1 , la curva directriz. es dado por: D :  z  0 Sea P '  x ', y ', z '  el punto de intersección de la directriz entonces la  x '2  y '2  1 D: , ...(1) z '  0  Calculando la ecuación de la generatriz x  x' y  y' z G:   1 1 2 satisface:

68

De la ecuación (1) y (2) eliminamos los parametros x', y'. z   x  x '  2 z  y  y'  z  2

2x  z   x '  2  y '  2y  z  2

2x  z 2 2 y  z 2 ) ( )  1, desarrolando se 2 2 tiene: 2 x 2  2 y 2  z 2  2 xz  2 yz  2

reemplazando (3) en (1) se tiene (

12. Hallar la ecuación de la superficie cónica cuya directriz es la elipse 4x2 + z = 1,y = 4 y cuyo vértice es el punto V(1,1,3) SOLUCION La ecuación directriz es:

4 x 2  z 2  1 D: como p'(x',y'.z')  D, y  4 4 x '2  z '2  1 entonces D :  y  4

La ecuación de la directriz es: G :

x 1 y 1 z  3   x ' 1 3 z ' 3

...(1)

...(2)

De (2) despejamos los parámetros x’, y’, z’:

 x 1  x ' 1    z 3   z ' 3

4(

y 1 3 y 1 3



3x  y  4  x '  y 1    z '  3 y  3z  12  y 1

3x  y  4 2 3 y  3z  12 2 ) ( )  1 de donde y 1 y 1

36 x 2  12 y 2  9 z 2  24 xy  18 xz  96 x  102 y  72 z  207 13. El eje OZ es el eje de un cono circular que tiene el vértice en el origen de coordenadas; el punto M(3,-4,7) está situado en su superficie. Hallar la ecuación de este cono.

SOLUCIÓN 69

Sea A(0,0,7), M(3,-4,7)

MA  A  M   3, 4,0  R || MA || 9  16  5 es el radio de la sección

circular del cono. Si z = 7, entonces la directriz es:

 x2  y 2  2  x '2  y '2  25 Sea D :  pero como p'(x',y',z')  D entonces: D :  ...(1) z  7 z '  7 Ahora calculamos la ecuación de la generatriz

G:

x0 y 0 z 0 x y z   , de donde G :   ...(2) x ' 0 y ' 0 z ' 0 x' y' 7

De la ecuación (2) despejamos x’, y’ se tiene: x z  x '  7 y z    y ' 7



7x  x '   z  7 y'  y  z

...(3)

Ahora reemplazando (3) en (1) se tiene.

(

7x 2 7 y 2 )  ( )  25 de donde 49x 2  49 y 2  25 z 2 z z

14. Hallar la ecuación de la superficie cuya directriz es la parábola x2 = 4y, z = 0, contenida en el plano XY, y cuya directriz tiene por primeros directores [1,1,3]. SOLUCIÓN

 x2  4 y La ecuación de la directriz es: D :  z  0 Sea p'(x',y',z') punto de intersección de la directriz y la generatriz entonces satisface a la  x '2  4 y ' ecuación de la directriz: D :  ...(1) z '  0  La ecuación de la directriz que pasa por el punto p'(x',y',z') con números directores 1,1,3 es:

70

x  x' y  y' z  z' z   ,G : x  x '  y  y '  ...(2) 1 1 3 3 Ahora eliminamos los parámetros (1) y (2)

G:

z 3x  z   x '  x  3  3   ...(3)  y '  y  z  3y  z 3 3  3x  z 2 3y  z 2 Re emplazando (3) en (1): ( )  4( ) 3 3  9 x 2  z 2  6 xz  36 y  12 z  0 z   x  x '  3  y  y'  z 3 

15. Demostrar que la ecuación x2  y 2  2 z 2  2 xz  2 yz  1 representa a una superficie cilíndrica; hallar las ecuaciones de su directriz y los números directores de su generatriz. SOLUCIÓN Consideremos las secciones paralelas al plano coordenado XY, z = k, obteniendo:

x2  y 2  2k 2  2kx  2ky  1. Completando cuadrado ( x  k )2  ( y  k )2  1 Luego ( x  k )2  ( y  k )2  1 , z = k, es una familia de circunferencia caso particular k=0, se tiene la ecuación de la directriz x2  y 2  1, z  0 que es una circunferencia en el plano XY por lo tanto

x2  y 2  2 z 2  2 xz  2 yz  1 es una superficie cilíndrica circular. La recta que une los centros (-k, k, k) de las circunferencias es paralela a la generatriz, por los tanto los números directores de la generatriz son  1,1,1 . Luego el grafico es:

16. Hallar la ecuación de la superficie generada por la rotación de la hipérbola

y 2  4 x2  4, z  0, entorno al eje Y SOLUCION La generatriz es de la forma x = f(y), z = 0 y el eje de rotación es el eje Y, por lo tanto la ecuación

de

la

y 2  4x2  4  x2 

superficie

de

revolución

es:

x2  z 2  f 2 ( y),

como

y2  4 y2  4  f 2 ( y ) ahora reemplazando se tiene: x 2  z 2  de 4 4

donde 4 x2  4 z 2  y 2  4  0 71

17. Hallar la ecuación den la superficie engendrada por la rotación de de la elipse.  x2 y 2 1   D :  a 2 b2 , alreddedor del eje OX z  0 

Consideremos un punto arbitrario en el espacio M ( x, y, z ) y c es pie de la perpendicular del

punto M al eje OX. El punto M lo trasladamos al plano OXY, mediante una rotación de esta perpendicular alrededor del eje OX designemos a este punto

N ( x ', y ',0) . Luego se tiene CM  CN , de donde CM  y 2  z 2

|y'|  y 2  z 2 CN  y ' entonces se tiene:  ...(1)  x '  x M está situado en la superficie de revolución, si y solo si, el unto N está en la elipse dada es decir:

x ' 2 y '2  1 a 2 b2

...(2)

ahora reemplazamos (1) en (2) tenemos:

x2 y 2  z 2   1, que es la ecuación de la a2 b2

superficie de la revolción buscada. 18. Discutir y graficar la superficie z 

2x x  y2 2

SOLUCIÓN -

Intersecciones con los ejes coordenados: x2 + y2 ≠ 0  x ≠ 0,y ≠ 0 o Con el eje X, se hace y = z = 0;  o Con el eje Y, se hace x = z = 0;  72

o Con el eje Z, se hace x = y = 0;  -

Las trazas sobre los planos coordenados o Sobre el plano XY, se hace z = 0; x = 0, y  R o Sobre el plano XZ, se hace y = 0; z 

2 x

o Sobre el plano YZ, se hace x = 0; z = 0

-

Simetrías o En el origen, existe o En los ejes coordenados,  eje X,  eje Y,  eje Z o En los planos coordenadas,  plano XY,  plano XZ,  plano YZ

-

Secciones transversales o En el plano XY; se hace z = k, obteniéndose k(x2 + y2) = 2x, familia de

1 1 circunferencias de centro ( , 0) y radio r  , k  0. k k

19. Discutir la grafica de la superficie z  ln  x 2  y 2  SOLUCIÓN -

Intersecciones con los ejes coordenados o Con el eje X, se hace y = z = 0; x = ± 1 o Con el eje Y, se hace x = z = 0; y = ± 1 o Con el eje Z, se hace x = y = 0;  z  R

-

Las trazas sobre los planos coordenados o Sobre el plano XY, se hace z = 0; ln  x 2  y 2   0 de donde x 2  y 2  1 circunferencia

73

o Sobre el plano XZ, se hace y = 0; z  2ln | x | o Sobre el plano YZ, se hace x = 0; z  2ln | y | -

Simetrías o En el origen  o En los ejes coordenados,  eje X,  eje Y,  eje Z o En los planos coordenadas,  plano XY,  plano XZ,  plano YZ

-

Secciones transversales o En el plano XY; se hace z = k, de donde ln  x 2  y 2   k  x 2  y 2  ek familia de circunferencias

20. Discutir y graficar la superficie |x| + |y| = 1 SOLUCIÓN Intersecciones con los ejes coordenados Con el eje X, se hace y = z = 0; x = ± 1 Con el eje Y, se hace x = z = 0; y = ± 1 Con el eje Z, se hace x = y = 0;  z  R Las trazas sobre los planos coordenados Sobre el plano XY, se hace z = 0; x  y  1 es un rombo Sobre el plano XZ, se hace y = 0; x = ± 1 Sobre el plano YZ, se hace x = 0; y = ± 1 Simetrías En el origen  En los ejes coordenados,  eje X,  eje Y,  eje Z En los planos coordenadas,  plano XY,  plano XZ,  plano YZ 74

Secciones transversales En el plano XY; se hace z = k, x  y  1

21. Discutir y graficar la superficie cuya ecuación es dada por x2  y 2  4 z  0 SOLUCIÓN Intersecciones con los ejes coordenados Con el eje X, se hace y = z = 0; x = 0 Con el eje Y, se hace x = z = 0; y = 0 Con el eje Z, se hace x = y = 0; z = 0 Las trazas sobre los planos coordenados Sobre el plano XY, se hace z = 0; x2  y 2  0 es un punto (0,0) Sobre el plano XZ, se hace y = 0; 4 z  x2 es una parábola Sobre el plano YZ, se hace x = 0; 4 z  y 2 es una parábola Simetrías En el origen  En los ejes coordenados,  eje X,  eje Y,  eje Z En los planos coordenadas,  plano XY,  plano XZ,  plano YZ Secciones transversales En el plano XY; se hace z = k, x2  y 2  4k , familia de circunferencias

22. Trazar la superficie cuya ecuación es x2  y 2  2 z 2  2 x  1 SOLUCIÓN

75

( x  1) 2 y 2   z2  1 2 2 es un hiperboloide de dos hojas de centro en (-1,0,0) x 2  y 2  2 z 2  2 x  1, completando cuadrados

su intersección en el eje x es -1+ 2, -1- 2, haciendo el traslado del origen (0,0,0)al punto C(-1,0,0) se tiene

23. Hallar la ecuación de la superficie engendrada por la rotación de la elipse  y2 z2  2  2 1 entorno del eje OY c b x  0 

SOLUCIÓN Consideremos un punto arbitrario del espacio M(x,y,z) y que C es el pie de la perpendicular bajada del punto M al eje OY al punto M lo trasladamos al plano OYZ mediante una rotación de esta perpendicular alrededor del eje OY y a este punto designamos por N(0,y,z) ahora haremos el dibujo correspondiente a la superficie, mediante el cual daremos la ecuación de dicha superficie.

76

CM  CN donde CM  x 2  z 2 , CN  z1 de donde z1  x 2  z 2 ...(1) además es evidente que y1  y

...(2)

El punto M(x,y,z) está situado en la superficie de revolución si y solo si y12 z12  1 ...(3) b2 c2 y 2 x2  z 2 de las igualdades (1) y (2) en (3) se tiene 2   1 que es la ecuación buscada b c2 N(o, y1 , z1 ) estáen la elipse dada, es decir:

24. Sean E : x2  y 2  z 2  2 x  2 y  10 z  29 y la recta

L   6, 10, 4   t  3,5, 4  / t  R . Hallar la ecuación de la superficie cilíndrica cuyos números directores de las generatrices resultan al efectuar el producto vectorial de los vectores normales a los planos tangentes a la esfera E en el punto de intersección de este cilindro es la curva que resulta de interceptar la esfera con el plano XZ SOLUCION

E : x 2  y 2  z 2  2 x  2 y  10 z  29, completando cuadrados E :  x  1   y  1   z  5  56 donde C  1,1  5  centro de la esfera: 2

2

2

L   6, 10, 4   t  3,5, 4  / t  R   6  3t , 10  5t , 4  4t  / t  R Sea P  L  E entonces P  L  P  E de donde Si P  L  P  6  3t , 10  dt , 4  4t  para algun t  R Como P  E   5  3t    11  5t    9  4t   56, de donde 2

2

2

t1  2, t2  3  para t1 =2, P1 1, 1,1 ; y para t2  3, P2  4, 4  3 Los vectores normales a los planos tangentes son: 1  CP1  P1  C  (2, 2,6), 2  CP2  P2  C  (5,3, 2) calculando el producto vectorial de las normales a los planos tangentes

i j k 1 x 2  2 2 6  (22, 26,16) 5 3 2 La curva directriz resulta de interceptar la esfera E con el plano XZ entonces y = 0por lo

( x  1)2  ( z  5)2  55 la curva directriz tanto: D :  y  0

77

Sea P’(x’,y’,z’) un punto de la intersección de la directriz con la generatriz, entonces la

( x ' 1) 2  ( z ' 5) 2  55 D: y'  0

satisface:

Calculando la recta generatriz del cilindro G :

G:

...(1)

x  x' y  y' z  z' de donde:   22 26 16

x  x' y  y' z  z'   11 13 8

...(2)

de la ecuación (1) y (2) eliminamos los parámetros x’,z’, de la ecuación (2) se tiene:

x  x' y  11  13  z  z'  y 13  8



11y   x '  x  13   y '  z  8y 13 

...(3)

ahora reemplazamos (3) en (1) tenemos:

(x 

11y 8y  1)2  ( z   5) 2  55, desarrolando se tiene: 13 13 169 x2  198 y 2  169 z 2  286 xy  208 yz  338x  754 y  1690 z  5070

25. Hallar la ecuación de la superficie engendrada por la rotación de la hipérbola

x2 z 2   1, y  0, alrededor del eje OZ a2 c2 SOLUCIÓN Sean M(x,y,z) un punto en el espacio tomado arbitrariamente, y D el pie de la perpendicular trazada desde el punto M al eje OZ. El punto M lo trasladamos al plano OXZ mediante una rotación de esta perpendicular al eje OZ. Designemos este punto en dicha situación por N(x’,0y’). Luego || DM |||| DN || donde || DM || ( x  0)2  ( y  0)2  ( z  z )2  x 2  y 2

Además || DN ||| x ' | por lo tanto | x ' | x 2  y 2 además z  z '

...(1)

El punto M esta situado en la superficie de revolución si y solamente si, el punto N está en la hipérbola dada, es decir: si

x '2 z '2  1 a2 c2

...(2)

78

Ahora reemplazamos (1) en (2) se tiene

( x 2  y 2 )2 z 2  2  1 simplificando a2 c

x2  y 2 z 2  2  1, ecuación de la superficie engendrada. a2 c 26.- Una esfera tiene el centro en la esfera: 2 x  4 y  z  7  0  L:  4 x  5 y  z  14  0

y es tangente a los planos:  1 : x  2 y  2 z  2 Hallar su ecuación: Solución

,

 2 : x  2 y  2 z  4 . P1

Como el centro C(h,k,l) de la esfera está en L  .........(1) 2 h  4 k  l  7  0  ..........(2)  4 h  5 k  l  14  0 resolviendo estas dos ecuaciones se tiene: 3k  7 de (1) + (2) se tiene : h  2

C(h,k,l)

P2

luego (2) – 2 (2) se tiene que k = l Ahora como

 d1 







d1 C, P1  d 2 C, P2

h  2k  2l  2



d2 

h  2k  2l  4

........................(3) 3 3 de esto tenemos que h = -1 entonces k = l = 3 en (3) se tiene el radio r =1 y el centro C(h,k,l) = ( -1, 3 , 3 ). Luego la ecuación de la esfera será:

x  1  y  3  z  3 2

2

2

1

27.- Hallar la ecuación de la esfera de radio 3 que es tangente al plano:  : x  2 y  2 z  3  0 en el punto M (1 , 1 ,- 3 ). Solución Si:  : x  2 y  2 z  3  0 su vector normal  De este plano es n 1 , 2 , 2 En la figura se observa que la ec. de la recta es: x 1 y 1 z  3 L:   de esto se tiene que: 1 2 2 k=2h–1 , l=2h–5 ...............(1) Luego la esfera que buscamos es: 2 2 2 E : x  h  y  k  z  l  9 ............(2)





 

M(1,1,- 3)



 

C(h,k,l)



79



 

 



Como M E E : 1  h  1  k   3  l  9 ...(3) Reemplazando (1) en (3) se tiene que h1 = 0 y h2 = 2 Si h1 = 0 entonces k1 = 1 ; l1 = - 5  E1 : x 2  y  1 2  z  5 2  9 Si h2 = 2 entonces k2 = 3 ; l1 = - 1  E2 : x  2 2  y  3 2  z  1 2  9 2

2

2





      



28.- Hallar las ecuaciones de los planos tangentes a la esfera E: x 2  y 2  z 2  10 x  2 y  26 z  113 y paralelos a las rectas:

x  5 y  1 z  13   2 3 2

L1 :

,

x  7 y 1 z  8   3 2 0

L2 :

Solución

P1

Completando cuadrados la esfera dada se tiene: E : x  5 2  y  1 2  z  13 2  308  de la recta L1 se obtiene su dirección a  2 , 3 , 2  y la recta L2 se obtiene su dirección b  3 , 2 , 0 Luego como las rectas dadas son paralelas a los Planos tangentes buscados el vector normal de estos



 

 









i j k    planos es: n  a  b  2  3 2  4, 6, 5 3 2 0





C(h,k,l)

 P2

De la figura dada se observa que la recata L pasa Por el centro de la esfera dada cuyo centro es: C = (5 , - 1 , - 13 ) cuya ec. es:

x  5 y  1 z  13    t cuya ecuación paramétrica es: 4 6 5 L : 4 t  5 , 6 t  1 , 5 t  13 ..........( * ) , L:





Ahora interceptamos la recta L con la esfera E dada y se tiene: L  E  4 t  5  5 2  6 t  1  1 2  5 t  13  13 2  308 de esto tenemos el valor de t  2 que reemplazado en ( * ) Para t = 2 se tiene P1 (13, 11, - 3) Para t = - 2 se tiene P2 (-3 , -13, -23)  Entonces como la ecuación del plano es n P  P1  0 . Ya conocemos el vector  normal : n  4, 6, 5



 



 







 





 



Para: P1 (13, 11, - 3)  4, 6, 5 x, y, z  13, 11,  3  0 . Operando se tiene:  1 : 4 x  6 y  5 z  103  0 Para: P2 (-3, - 13, - 23)  4, 6, 5 x, y, z   3,  13,  23  0 . Operando se tiene:



 

 



80

 2 : 4 x  6 y  5 z  205  0 29.- El punto C (1,-1,-2) es el centro de una circunferencia que corta en la recta:  2 x  y  2 z  12  0 ; una cuerda de longitud igual a 8. Hallar la ecuación L:  4 x  7 y  z  6  0 de esta circunferencia. Solución. La recta dada está en forma biplanar, resolviendo el sistema de los dos planos que se interceptan para dar origen a la recta dada se tiene: 2 x  y  2 z  12  0 ..........(1) , de esto 4x 7 y  z  6  0 ........(2) hacemos (2) – 2(1) se tiene: 3 y  6 z, x  z  9 ,haciendo z = t 2 se tiene la recta L en forma vectorial: C(h,k,l)   3    L :  9,6,0  t  ,  1, 1 / t  R     2    De la figura A y B pertenecen a la recta L





 3  A   9   ,6   ,  2 

  /  ;   2

 3 B   9  r , 6  r , r 2 

 3 3  AB  8   9    9  r   6    6  r 2 2  



 

  r    2

2



9 2 2 2 r  r  r  4 64  4 2 .................................................................(1) r  17

 64 



 



  / rR  



2

  3 2 Se sabe que: R   9    1  6    1    2 2  



2

 



2

........(2)

  3 2 2 También que: R   9  r  1  6  r  1  r  2 ........(3) 2   De que (2) = (3) r    8  r    Si r   reemplazando en (1) se tiene que es falso 8 17  Si r    8 en (1) se tiene que   4  ................(4) 17 8 17 (4) en r    8 se tiene que: r  4  .....................(5) 17  12 17 8 17 8 17  8 17 ,2  ,4  para:   4  en A se tiene A  3   17 17 17  17  



 



81

Luego el radio es la distancia de A al centro C que desarrollando se tiene el radio es 2 2 2 r 2  65  C 1,1,2  E : x  1  y  1  z  2  65 Para encontrar la ecuación de la circunferencia nos falta la ecuación del plano que intercepta a la esfera E para esto ahora encontramos el valor de B, tomamos el 8 17 valor de r  4  17  12 17 8 17 8 17   B  9  ,2  ,4   17 17 17       : n P  P0  0 pero n  CB  CA









 

 





i  12  n  2 17 12 2 17

j 3 3

k 8

6

17 8

6

17

  144 176  40    , ,  17 17 17   17 8 8

17

   144 176  40   17  n , ,    18, 22,  5  17   8  17 17     Luego la ecuación del plano es:









 :  18, 22,  5 x, y, z   1,  1,  2   0   : 18 x  22 y  5 z  30  0 Por lo tanto la ecuación de la circunferencia es: 2 2 2  E : x  1  y  1  z  2  65 C:   : 18 x  22 y  5 z  30  0 30.- Hallar la ecuación del plano que pasa por la línea de intersección de las dos esferas:



 

 



E1 : 2 x 2  2 y 2  2 z 2  3 x  2 y  z  5  0 E2 : x 2  y 2  z 2  x  3 y  2 z  1  0 Solución De las esferas dadas obtenemos el plano restando:

E1  2 E2 : 5 x  8 y  5 z  7  0 ......................(*) (*) viene hacer la ecuación del plano radical de las esferas dadas.

82

x  2 y 1 z 1 es el eje de un cono circular cuyo vértice está   2 2 1 situado en el plano OYZ. Hallar la ecuación de este cono, si se sabe que el punto   5  está situado en la superficie. P1, 1, 2   Solución Paremos la recta dada en una ecuación paramétrica: x  2 y 1 z 1 L:   t 2 2 1 x  2 t  2  L :  y  2 t  1 , Como el vértice pertenece a la recta L  z  t  1  y V   : OYZ  V 0, a, b  V x, y, z  x  0  2 t  2  x  0  t  1 Luego el vértice será V (0,1,0) 31.- La recta L :







 





L : 2,1,1  t 2,2,2 / t  R









donde el punto es Q  2,1,1 y la dirección de la  recta es: a  2,2,1





Ahora por el punto P trazamos un plano perpendicular al eje del cono, el cual interceptado con el cono nos dará una circunferencia que es la directriz de este cono; se sabe que: r  PC . Además C  L





 C  2  2 ,  1  2  ,  1   /   R ......................(1)   3  Además QP    1, 2, 2   QC   2,  2 ,  1











  QP  a     1,2, 3 2 2,2,1   Como QC  Pr oy  QP  a  2,  2,  1 a  a  9     1 1  QC  2,2,1   2,  2,  1    ............(2) 2 2 remplazando (2) en (1) obtenemos C  1, 0,  1 2











 



 



 PC  C  P  1, 0,  1 2  1 ,1,  5 2  0,  1, 2

 





 r  PC  0 2   1  2 2  5  Luego la ecuación del plano es:  : n P  P1  0 2





83

 x, y, z   1, 1,  5 2   0 que desarrollando se tiene :  : 4 x  4 y  2 z  5  0 y como C  1, 0,  1 2 y el radio r  5 Entonces la esfera es: E : x  1  y  z  1 2  5 

  Como n a   : 2,  2,  1

2

2

2

Luego la ecuación de la directriz será: 2 2 2  E : x  1  y  z  1 2  5 D:   : 4 x  4 y  2 z  5  0 Además como su vértice es V (0,1,0) tenemos la generatriz: x y 1 z .............................................................................(3) G:   x y   1 z 

















2 2 2  E : x  1  y   z   1 2  5 Como P  D entonces: D  :  ..............(4)     : 4 x  4 y  2 z  5  0   9x De (3) y (4) se obtiene que: x   4x4y2z 4 4x5y2z 5 9z y , z 4x4y2z 4 4x4y2z 4 Luego por eliminación de parámetros reemplazando estos valores en la ec. de 2do. Orden de la ec(4) se tiene la ec. del cono: 35 x 2  35 y 2  52 z 2  232 x y  116 x z  116 y z  232 x  70 y  116 z  35  0



 32.- Hallar la ecuación del plano que es perpendicular al vector a  2,  1,  2 2



2

x y   2z 3 4 Solución Sea P0  ( x0 , y0 , z 0 ) punto de tangencia entre  el paraboloide y el plano tangente  : n P  P0  0 y tangente al paraboloide elíptico:





x2 y2  2z  0 3 4 Se sabe que el vector gradiente  F  Fx , Fy , Fz es paralelo al vector normal   del plano tangente tal que: F n a Se sabe que: F(x,y,z) =









2x y    F  Fx , Fy , Fz   0 , 0 ,2    a 3  3  2x y    0 , 0 ,2    2,  1 ,2    1  x0  3 , y 0  2 3  3  x02 y 02   2 z 0 reemplazando los valores hallados en esta ecuación se Pero P0  S  3 4 tiene que z 0  2 se tiene la ec. del plano:









 : 2,  1,  2 x, y, z   3,2,2  0 o sea  : 2 x – y –2 z – 4 =0

84





x2 y2 z 2    1 corta a los ejes a2 b2 c2 a2 y2 z2 coordenados en L, M y N. Demostrar que el área del triángulo LMN es: , 2 p x1 y1 z1 donde p es la longitud de la perpendicular desde el origen hasta el plano tangente. Solución 33.- El Plano tangente en P1 x1 y1 , z1 al elipsoide





El Plano tangente al elipsoide en el punto P1 x1 y1 , z1 x x y y z z Es: PT : 1 2  1 2  1 2  1 a b c Ahora interceptamos este plano tangente con los ejes coordenados para hallar los puntos L,M,N . a2  Con el eje x: y  z  0  x  x1 

b2 Con el eje y: x  z  0  y  y1



c2 Con el eje z: x  y  0  z  z1

 a2   b2   c2 De esto se tiene que: L  , 0, 0  , M  0 , , 0  , N  0 , 0 ,  x1   y1   z1      Luego se tiene que el área del triángulo es:

A L M N 

1 2p

a2 x1

0

0

0

b2 y1

0 

0

0

c2 z1

P1

   

a2 b2 c2 , donde p es la longitud de la 2 p x1 y1 z1

perpendicular desde el origen hasta el plano tangente. Nota: Cuando no se sabe cuál es la ec. del plano tangente al elipsoide en P1 se deduce  de la ec. n P  P1  0 . Se sabe que el vector gradiente  F  Fx , Fy , Fz es paralelo al vector normal 34.- Hallar la transformación de la ecuación 23 x 2  41 y 2  31 z 2  48 x y  72 x z  24 y z  0 al hacer girar los ejes coordenados de tal manera que los cosenos directores de los nuevos ejes con respecto a los originales sean: 2 3 6  6  2 3 3  6 2  , ,  ,     7 7 7  7 , 7 , 7 , 7 , 7 , 7       Solución Las ecuaciones de transformación de las originales a las nuevos ejes son:









85

 x  x  cos  1  y  cos  2  z  cos  3  T :  y  x  cos  1  y  cos  2  z  cos  3 .....................(1)  z  x  cos   y  cos   z  cos  1 2 3  de donde: 2 3 6 cos  1  , cos  1  , cos  1  7 7 7 6 2 3 cos  2  , cos  2  , cos  2  .............................(2) 7 7 7 3 6 2 cos  3  , cos  3  , cos  3  7 7 7 2 2 2 23 x  41 y  31 z  48 x y  72 x z  24 y z  0 ........................(3) Reemplazando (2) en (1) obtenemos:

 2 6 3 y   z  ..............( A)  x  x  7 7 7  3 2 6  T :  y  x   y   z  ..................( B) ......................................(4) 7 7 7  6 3 2   z  7 x   7 y   7 z  ..................(C )  Ahora reemplazamos (4) en (3) obtenemos: 23 A2  41 B 2  31 C 2  48 A B  72 A C  24 B C  0 ......................(5) desarrollando (5) obtenemos:  2401 x 2  2401 y 2  2401 z 2  0  x2  y2  z 2  0 el cual es un cono circular recto.

86

EJERCICIOS DE APLICACIÓN

1.- Discutir y graficar. 8 x  4 x y  5 y  z Sugerencia: Usar la transformación 2

2

2

 72

 x  x cos   y  sen T :  y  x sen  y  cos 

x2 y2   z 2  1 a sus puntos 3 2 2 2 de intersección con el cilindro: 16 x  9 y  54 , intersectan al plano z = 0 en puntos sobre un circulo cuya ecuación es : 2 x 2  2 y 2  3 .

2.- Probar que las normales a la elipsoide:

3. Las generatrices de un cilindro circunscrito en la esfera: x 2  y 2  z 2  2 x  4 y  2 z  3  0 , son paralelas a la recta: x  t  3  L :  y  t  7 .  z  2 t  5 

Hallar la ecuación de este cilindro. 4. Hallar la ecuación del cono que tiene el vértice en el origen de coordenadas, si las generatrices son tangentes a la esfera:

x  2 2   y  1 2  z  3 2  9

5. Hallar la ecuación del cilindro circunscrito en las esferas: E1 : x  5    y  1  z  2  16 2

2

2

,

E1 : x  3    y  1  z  1  16 2

2

2

6. Los puntos A(3,-2,5); B(-1,6,-3) son los extremos de un diámetro de una circunferencia que pasa por el punto C(1,-4,1). Hallar la ecuación de esta circunferencia 7. Identificar que superficie representa la ecuación:

x2  2 y2  3 z 2  4 y z  2 x z  2  0 . Determine sus elementos. 8. Encuentre la ecuación del cono cuya generatriz pasa por los puntos (0,2,2) y (0,3,4) y su eje es el eje z.

9. Discutir y graficar: 16 x 2  9 y 2  z 2  144

87

10. Hallar la ecuación paramétrica del diámetro de la esfera: E  x 2  y 2  z 2  2 x  6 y  z  11  0 que es perpendicular al plano

 :5 x  y  2 x  17  0

11. Hallar la ecuación de la circunferencia que pasador los tres puntos: M1(3,-1,-2) ,

M2(1,1,-2) , M3(-1,3,0).

12. Encuentre la ecuación del cono cuya generatriz pasa por los puntos (0,2,2) y (0,3,4) y su eje es el eje z. 13. El Plano tangente en P1 x1 y1 , z1 

x2 y2 z 2 al elipsoide 2  2  2  1 corta a a b c

los ejes coordenados en L, M y N. Calcular el área del triángulo LMN, y se tiene p es la longitud de la perpendicular desde el origen hasta el plano tangente al elipsoide.

14. Hallar la ecuación del cilindro circular que pasa por el punto S(2,-1,1), si la recta: x = 3 t +1 ,

y=-2t-2

, z = t + 2, es eje del mismo

15. ¿En qué punto de la elipsoide: E :

x2



y2

 z 2  1 . La normal trazada

4 4 a ésta forma ángulos iguales con los ejes coordenados. 16. Determinar el centro C y el radio R de la circunferencia:

x  3  2   y  2  2  z  1  2  100   2x2 y z  9  0

88

Get in touch

Social

© Copyright 2013 - 2024 MYDOKUMENT.COM - All rights reserved.